exam #2 musculoskeletal part medsurgII

Réussis tes devoirs et examens dès maintenant avec Quizwiz!

patient with a joint dislocation is educated regarding proper exercises and activities as well as danger signs and symptoms to look for, such as increasing pain (even with analgesic agents), numbness or tingling, and increased edema in the extremity.These signs and symptoms may indicate __________________

compartment syndrome

dislocation

complete separation of joint surfaces

- grating or crackling sound or sensation - may occur with movement of ends of a broken bone or irregular joint surface

crepitus

Identify at least five musculoskeletal problems that can cause acute low back pain

cute lumbosacral strain,unstable lumbosacral ligaments, weak lumbosacral muscles, osteoarthritis ofthe spine, spinal stenosis, intervertebral disc problems, and unequal leg length

A patient had hand surgery to correct a Dupuytren contracture. What nursing intervention is a priority postoperatively? a. Changing the dressing b. Applying a cock-up splint and immobilization c. Having the patient exercise the fingers to avoid future contractures d. Performing hourly neurovascular assessments for the first 24 hours

d. Performing hourly neurovascular assessments for the first 24 hours

Osteomyelitis with vascular insufficiency, which most commonly affects the feet, is seen most often among patients with ________ and ________.

diabetes, peripheral vascular disease

Tinel sign

direct percussion of the median nerve at the wrist produces no symptoms in the normal hand. If positive test, percussion produces tingling and burning along distribution. Sign of carpal tunnel syndrome

cortical bone

hard, dense, strong bone that forms the outer layer of bone; also called compact bone

malunion

healing of a fractured bone in a malaligned position

examples of long bones

humerus, radius, ulna, femur, tibia, fibula

area where bone ends meet; provides for motion and flexibility

joint

An effusion is suspected if the joint is swollen and the normal bony landmarks are obscured. The most common site for joint effusion is the _________

knee

Single ___ or hip ____ are most commonly infected in patients with septic arthritis, although up to 20% of cases involve more than one joint (i.e., polyarticular disease)

knee joints

increase in the convex curvature of the thoracic spine

kyphosis

abnormal sensation (e.g., burning, tingling, numbness)

paresthesia

delayed union

prolongation of expected healing time for a fracture

PRICE acronym

protection, rest, ice, compression, elevation

A patient presents to a clinic complaining of a leg ulcer that isn't healing; subsequent diagnostic testing suggests osteomyelitis. The nurse is aware that the most common pathogen to cause osteomyelitis is what? A) Staphylococcus aureus B) Proteus C) Pseudomonas D) Escherichia coli

Ans: A Feedback: S. aureus causes over 50% of bone infections. Proteus, Pseudomonas, and E. coli are also causes, but to a lesser extent.

Dupuytren's contracture causes flexion of which area(s)? a) Fourth and fifth fingers b) Ring finger c) Index and middle fingers d) Thumb

Fourth and fifth fingers Explanation: Dupuytren's contracture causes flexion of the fourth and fifth fingers, and frequently the middle finger.

depressed fracture

Fragments are driven inward

One side of a bone is broken and the other side is bent

Greenstick

Which of the following was formerly called a bunion? a) Ganglion b) Plantar fasciitis c) Morton's neuroma d) Hallux valgus

Hallux valgus Explanation: Hallux valgus (commonly called a bunion) is a deformity in which the great toe deviates laterally. Plantar fasciitis, an inflammation of the foot-supporting fascia, present as an acute onset of heel pain experienced with the first steps in the morning. Morton's neuroma is a swelling of the third (lateral) branch of the median plantar nerve. A ganglion, a collection of gelatinous material near the tendon sheaths and joints, appears as a round, firm compressible cystic swelling, usually on the dorsum of the wrist

Which is a deformity in which the great toe deviates laterally? a) Plantar fasciitis b) Hammertoe c) Pes cavus d) Hallux valgus

Hallux valgus Explanation: Hallux valgus is a deformity in which the great toe deviates laterally. A hammertoe is a flexion deformity of the interphalangeal joint, which may involve several toes. Pes cavus refers to a foot with an abnormally high arch and a fixed equines deformity of the forefoot. Plantar fasciitis is an inflammation of the foot-supporting fascia.

A patient with an arm cast reports pain in the extremity. What is a priority nursing action to reduce the incidence of complications? (Select all that apply.) a. Assess the fingers for color and temperature b. Administer a prescribed analgesic to promote comfort and allay 376 anxiety c. Assess for a pressure sore d. Determine the exact site of the pain e. Cut the cast with a cast saw

a, c, d

-dronate

bisphosphonate; bone resorption inhibitor

Bisphosphonates

Inhibit osteoclasts, causing decreased bone loss and increased bone mass

A client has Paget's disease. An appropriate nursing diagnosis for this client is: a) Risk for falls b) Delayed wound healing c) Fatigue d) Risk for infection

Risk for falls Explanation: The client with Paget's disease is at risk for falls secondary to pathological fractures and impaired gait/mobility.

A patient shows the nurse a round, firm nodule on the wrist. The pain is described as aching, with some weakness of the fingers. What treatment does the nurse anticipate assisting with? (Select all that apply.) a. Educating the patient on the use of gabapentin b. Active range-of-motion exercises c. Corticosteroid injections d. Surgical excision e. Aspiration of the cyst

c. Corticosteroid injections d. Surgical excision e. Aspiration of the cyst

Bone scan nursing interventions

- allergies to the radioisotope - assesses condition that would contraindicate performing the procedure (e.g., pregnancy, breast-feeding). - explain that patient may experience moments of discomfort from the isotope (e.g., flushing, warmth) but provide reassurance that the radionuclide poses no radioactive hazard - drink plenty of fluids to help distribute and eliminate the isotope. - Before empty the bladder

epicondylitis

- chronic, painful condition that is caused by excessive, repetitive extension, flexion, pronation, and supination motions of the forearm. - These motions result in inflammation (tendonitis) and minor tears in the tendons at the origin of the muscles on the lateral or medial epicondyles.

Secondary osteoporosis

- the result of medications or diseases that affect bone metabolism. - Men are more likely than women to have secondary causes of osteoporosis, including the use of corticosteroids and excessive alcohol intake.

Callus (foot)

- thickened area of the skin that has been exposed to persistent pressure or friction. - Faulty foot mechanics usually precede the formation of a callus. - Treatment consists of eliminating the underlying causes and having a painful callus treated by a podiatrist

Nursing Management of a Patient in a Cast, Splint, or Brace

-Swelling is a concern - before applying assess neurovascular status and skin - patient may require a tetanus booster - Assessments first 24 hours and every 1 to 4 hours thereafter to prevent neurovascular compromise related to edema and/or the device.

Joint dislocation can lead to avascular necrosis if it is not treated. What rationale should be provided to the patient with the potential for developing avascular necrosis?

AVN is tissue death due to anoxia and diminished blood supply.

A client is admitted with acute osteomyelitis that developed after an open fracture of the right femur. When planning this client's care, the nurse should anticipate which measure? a) Withholding all oral intake b) Instructing the client to ambulate twice daily c) Administering large doses of I.V. antibiotics as ordered d) Administering large doses of oral antibiotics as ordered

Administering large doses of I.V. antibiotics as ordered Explanation: Treatment of acute osteomyelitis includes large doses of I.V. antibiotics (after blood cultures identify the infecting organism). Surgical drainage may be indicated, and the affected bone is immobilized. The client usually requires I.V. fluids to maintain hydration, but oral intake isn't necessarily prohibited.

cartilaginous/fibrous tissue at fracture site

callus

The most effective cleansing solution for care of a pin site is ________.

chlorhexidine solution

rhythmic contractions of a muscle

clonus

The nurse is reviewing the medication administration record of the client. Which of the following medications would lead the nurse to suspect that the client is at risk for osteoporosis? a) methotrexate (Rheumatrex) b) penicillamine (Cuprimine) c) plicamycin (Mithracin) d) raloxifene (Evista)

raloxifene (Evista) Explanation: Raloxifene (Evista) is used for the prevention and treatment of osteoporosis.

débridement

removal of contaminated or dead tissue and foreign matter from an open wound

fracture reduction

restoration of fracture fragments into anatomic alignment

Joints can become infected through spread of pathogens from other parts of the body (hematogenous spread) or directly through trauma or surgical instrumentation, causing

septic arthritis.

diaphysis

shaft of long bone

how do patients describe fracture pain?

sharp and piercing and is relieved by immobilization.

amphiarthrosis

slightly movable joint

Dupuytren contracture of palmar fascia

slowly progressive contracture of the palmar fascia that causes flexion of the fourth, fifth, and, sometimes, middle finger, rendering these fingers more or less useless

Atonic muscle

soft and flabby

yellow marrow

soft, fatty material found in the medullary cavity of long bones

a partial dislocation and does not cause as much deformity as a complete dislocation

subluxation

Joint arthroplasty

surgical implantation of an artificial joint

total hip arthroplasty

surgical reconstruction of hip with artificial hip joint

There are three basic kinds of joints:

synarthrosis, amphiarthrosis, and diarthrosis joints.

cord of fibrous tissue connecting muscle to bone

tendon

Muscle tendon sheaths become inflamed with repetitive stretching, causing

tendonitis

how does primary provider assess for pressure ulcer development in a patient with a cast?

the primary provider may univalve, bivalve, or cut an opening (window) in the cast to allow for inspection, access, and possible treatment

cryotherapy

therapeutic use of cold

After skin traction is applied, the nurse assesses circulation of the foot within 15 to 30 minutes and then every 1 to 2 hours.

true

Osteomalacia may result from failed calcium absorption or from excessive loss of calcium from the body (e.g., kidney failure).

true

how do patients describe bone pain?

typically described as a dull, deep ache that is "boring" in nature. *This pain is not typically related to movement and may interfere with sleep.

The major defect in osteomalacia is a deficiency of activated ___________, which promotes calcium absorption from the gastrointestinal tract and facilitates mineralization of bone.

vitamin D

general term that describes impaired movement of the rotator cuff of the shoulder

Impingement Syndrome

-These injuries leave loose cartilage in the knee joint that may slip between the femur and the tibia, preventing full extension of the leg. -If this happens during walking or running, the patient often describes the leg as "giving way."

Meniscal Injuries

nerve palsy

Momentary loss of function caused when cold is applied to a part that has motor nerves close to the skin.

The nurse is caring for a patient postoperatively following orthopedic surgery. The nurse assesses an oxygen saturation of 89%, confusion, and a rash on the upper torso. What does the nurse suspect is occurring with this patient? a. Polyethylene-induced infection b. Pneumonia c. Fat emboli syndrome d. Disseminated intravascular coagulatio

Fat emboli syndrome

disarticulation

amputation through a joint

shortening of surrounding joint structures

contracture

sprain

injury to a ligament

new bone growth around a sequestrum

involucrum

fasciculation

involuntary twitch of muscle fibers

open reduction

involves surgical exposure of the bone and the use of plates, screws, or pins to realign the fragments

compartment syndrome

involves the compression of nerves and blood vessels due to swelling within the enclosed space created by the fascia that separates groups of muscles

Volkmann's ischemic contracture

is a permanent shortening (contracture) of forearm muscles, usually resulting from injury, that gives rise to a clawlike deformity of the hand, fingers, and wrist

The junction of two or more bones is called a __________, or articulation

joint

synovium

membrane in joint that secretes lubricating fluid

Those experiencing pain with a rheumatic disorder experience pain that is worse in the __________________

morning * especially upon waking.

isotonic contraction:

muscle is shortened without a change in its tension; a joint is moved as a result

isometric contraction

muscle tension is increased without changing its length; there is no associated joint motion

involucrum

new bone formation

6 P's of neurovascular assessment

pain, pulse,pallor,paresthesia, paralysis, pressure

red bone marrow function

produce red blood cells, WBC and platelets (HEMATOPOIESIS )

having greater-than-normal muscle tone

spastic

Lateral and medial collateral ligaments of the knee provide _______ to the knee

stability

A condition resulting from damaging a muscle or tendon

strain

A muscle tear that is microscopic and due to overuse is called a ________.

strain

Promote Shoulder Healing of Impingement Syndrome

- Rest the joint in a position that minimizes stress on the joint structures to prevent further damage - Support the affected arm on pillows while sleeping - Gradually resume motion and use of the joint. - Assistance with dressing and other activities of daily living may be needed. - Avoid working and lifting above shoulder level or pushing an object against a "locked" shoulder. - Perform ROMand strengthening exercises.

A client with osteoporosis is prescribed calcitonin (Miacalcin) 100 units subcutaneously. The medication is available 200 units per ml. How many milliliters will the nurse administer to the client?

0.5 Explanation: 100 units x 1 ml/200 units = 0.5 ml.

Meniscal Injuries

- Normally, little twisting movement is permitted in the knee joint. - Twisting of the knee or repetitive squatting and impact may result in either tearing or detachment of the cartilage from its attachment to the head of the tibia.

nurse can change pressure of pins in external fixation devices

False, the only person that can do that is the doctor

A client with osteoporosis is prescribed a selective estrogen receptor modifier (SERM) as treatment. The nurse would identify which drug as belonging to this class? a) Raloxifene (Evista) b) Calcium gluconate c) Tamoxifen (Nolvadex) d) Alendronate (Fosamax)

Raloxifene (Evista) Explanation: An example of a selective estrogen receptor modifier (SERM) is raloxifene (Evista). Alendronate is a biphosphonate; calcium gluconate is an oral calcium preparation; tamoxifen is an antiestrogen agent.

osteomyelitis manifestation

The patient may describe a constant, pulsating pain that intensifies with movement as a result of the pressure of the collecting purulent material (i.e., pus).

The nurse suspects that a patient with an arm cast has developed a pressure ulcer. Where should the nurse assess for the presence of the ulcer? a. Lateral malleolus b. Olecranon c. Radial styloid d. Ulna styloid

Ulna styloid

excessive fluid within the capsule

effusion

osteomalacia

metabolic bone disease characterized by inadequate mineralization of bone. *deficiency of activated vitamin D

A fracture occurs at an angle across the bone

oblique

Examination of synovial fluid is helpful in the diagnosis of

- septic arthritis and other inflammatory arthropathies and reveals the presence of hemarthrosis (bleeding into the joint cavity), which suggests trauma or a bleeding disorder.

name three early and three delayed complications of fractures.

-Early: shock, fat embolism,compartment syndrome, deep vein thrombosis, thromboembolism, DIC, andinfection. - Delayed: delayed union and nonunion, avascular necrosis of bone, reaction to internal fixationdevices, complex regional pain syndrome (CRPS), and heterotrophicossification.

An older adult female has a bone density test that reveals severe osteoporosis. What does the nurse understand can be a problem for this client due to the decrease in bone mass and density? a) Diabetes b) Cardiac disease c) Hypertension d) Compression fractures

Compression fractures Explanation: In osteoporosis, loss of bone substance exceeds bone formation. The total bone mass and density is reduced, resulting in bones that become progressively porous, brittle, and fragile. Compression fractures of the vertebrae are common. Diabetes, hypertension, and cardiac disease may occur in response to the aging process but are not the result of osteoporosis.

Tendon

Connects muscle to bone

The fractured bone is compressed by another bone

Compressed

Arthroscopy nursing considerations

- after procedure, joint is wrapped with a compression dressing to control swelling. - ice, to control edema and enhance comfort - kept joint extended and elevated to reduce swelling. - neurovascular status - avoid strenuous activity of the joint - monitor for signs and symptoms of complications (e.g., fever, excessive bleeding, swelling, numbness, cool skin)

Continuous Passive Motion (CPM)

- promote range of motion, circulation, and healing, and to prevent scar tissue from forming in the knee, which could decrease mobility and increase postoperative pain. * The patient's leg is usually placed in this device immediately after surgery

A patient is diagnosed with osteogenic sarcoma. What laboratory studies should the nurse monitor for the presence of elevation? a. Magnesium level b. Potassium level c. Alkaline phosphatase d. Troponin levels

Alkaline phosphatase

A group of students is studying for a test on traction. The students demonstrate understanding of the types of traction when they identify which of the following as an example of skin traction? a. Balanced suspension b. Thomas splint c. Crutchfield tongs d. Buck's

An example of skin traction is Buck's traction. Crutchfield tongs, a Thomas splint, or balanced suspension traction are types of skeletal traction.

The intervertebral discs that are subject to the greatest mechanical stress and greatest degenerative changes are ________, ________, and ________.

L4, L5, S1

_________ epicondylitis (i.e., tennis elbow) is frequently identified in someone who repeatedly extends the wrist with supination of the forearm. Pain develops over the lateral epicondyle and in the extensor muscles.

Lateral

The client presents to the emergency department with fever, chills, restlessness, and limited movement of a fractured jaw. The nurse interprets these findings as indicating which of the following complications? a) Osteomyelitis b) Fat embolism c) Avascular necrosis d) Compartment syndrome

Osteomyelitis Explanation: Clinical manifestations of osteomyelitis include signs and symptoms of sepsis and localized infection.

______________ is the most common and most often fatal primary malignant bone tumor. Prognosis depends on whether the tumor has metastasized to the lungs at the time the patient seeks health care.

Osteosarcoma

List four reasons for a patient to have traction application.

To minimize muscle spasms; to reduce, align, and immobilize fractures; to lessen deformities; and to increase space between opposing surfaces within a joint

how to prevent disuse syndrome?

To prevent this, the nurse instructs the patient to tense or contract muscles (e.g., isometric muscle contraction) without moving the underlying bone

Three medications used to treat Paget disease are ________, ________, and ________.

calcitonin, bisphosphonates, plicamycin

Normally, synovial fluid is

clear, pale, straw colored, and scanty in volume.

Pain associated with _________________________ is relentless and is not controlled by modalities such as elevation, application of ice or cold, and usual dosages of analgesic agents.

compartment syndrome

The earliest indicator of developing compartment syndrome?

pain that seems out of proportion to the underlying injury and pain on passive stretch of other muscles in the immobilized limb * cast, brace, or splint is too tight

carpal syndrome symptoms

pain, numbness, paresthesia, and, possibly, weakness along the median nerve distribution (thumb, index, and middle fingers). Night pain and/or fist clenching upon awakening is common.

subluxation

partial separation of joint surfaces

Ball-and-socket joints

permit full freedom of movement. (shoulder and hip)

Dorsiflexion of the foot demonstrates function of the ___________ nerve.

peroneal

A patient has a long-leg cast applied. Where does the nurse understand a common pressure problem may occur? a. Dorsalis pedis b. Peroneal nerve c. Popliteal artery d. Posterior tibialis

peroneal nerve

close reduction

position the bone in correct alignment and applying a case or splint to maintain the position

Contusions are managed with PRICE an acronym that refers to:

protection, rest, ice, compression, and elevation

cancellous bone

spongy bone, not as dense as compact bone

plantar flexion demonstrates function of the __________ nerve

tibial

The longest immobilization time necessary for fracture union occurs with a fracture of the ________.

tibial shaft

Phalen's sign

to assess carpal tunnel

examples of irregular bones

vertebrae and facial bones

Loose Bodies

• "Articular Mice" • Due to degeneration or trauma • S&S: Pain, effusion of the joint. - *Elbow locks and unlocks in various positions • TREATMENT: Removal through arthroscopic surgery

_____________ refers to a reduction in bone mass to below normal levels

osteopenic

Meniscus tear

A tear in the meniscus which is the cartilage that acts as a cushion for the joint

The most common complication of hip fractures in the older adult patient is________.

deep vein thrombosis

how do patients describe muscular pain?

described as soreness or aching and is referred to as "muscle cramps."

radiculopathy

disease of a spinal nerve root, often resulting in pain and extreme sensitivity to touch.

Paget Disease of the Bone

disorder of localized rapid bone turnover, most commonly affecting the skull, femur, tibia, pelvic bones, and vertebrae.

bursitis

inflammation of a fluid-filled sac in a joint

arthritis

inflammation of a joint

tendonitis

inflammation of a tendon

Osteomyelitis

inflammation of bone and bone marrow

sciatica

inflammation of the sciatic nerve, resulting in pain and tenderness along the nerve through the thigh and leg

The layman's term for onychocryptosis, a common foot condition, is ________.

ingrown toenail

A nurse is teaching a client who was recently diagnosed with carpal tunnel syndrome. Which statement should the nurse include? a) "Surgery is the only sure way to manage this condition." b) "Using arm splints will prevent hyperflexion of the wrist." c) "This condition is associated with various sports." d) "Ergonomic changes can be incorporated into your workday to reduce stress on your wrist."

"Ergonomic changes can be incorporated into your workday to reduce stress on your wrist." Explanation: Ergonomic changes, such as adjusting keyboard height, can help clients with carpal tunnel syndrome avoid hyperextension of the wrist. This condition is associated with repetitive tasks such as clerical work, not sports. The condition may be managed with medications, yoga, acupuncture, and wrist (not arm) splints.

Physical activity, particularly _______________________, acts to stimulate bone formation and remodeling.

weight-bearing activity

Bone Densitometry

x-ray technique for determining density of bone

During a routine physical examination on a 75-year-old female client, a nurse notes that the client is 5 feet, 3/8 inches (1.6 m) tall. The client states, "How is that possible? I was always 5 feet and 1/2? (1.7 m) tall." Which statement is the best response by the nurse? a) "After menopause, the body's bone density declines, resulting in a gradual loss of height." b) "After age 40, height may show a gradual decrease as a result of spinal compression" c) "The posture begins to stoop after middle age." d) "There may be some slight discrepancy between the measuring tools used."

"After menopause, the body's bone density declines, resulting in a gradual loss of height." Explanation: The nurse should tell the client that after menopause, the loss of estrogen leads to a loss in bone density, resulting in a loss of height. This client's history doesn't indicate spinal compression. Telling the client that measuring tools used to obtain the client's height may have a discrepancy or that the posture begins to stoop after middle age doesn't address the client's question.

Meniscectomy

- The most common site for meniscectomy is the knee - the procedure refers to the excision of damaged joint fibrocartilage.

The nurse has educated a patient with low back pain about techniques to relieve the discomfort and prevent further complications. What statement by the patient shows understanding of the education the nurse provided? a. "I will lie prone with my legs slightly elevated" b. "I will bend at the waist when I am lifting objects from the floor" c. "I will avoid prolonged sitting or walking" d. "Instead of turning around to grasp an object, I will twist at the waist"

"I will avoid prolonged sitting or walking"

The nurse is planning discharge instructions for the client with osteomyelitis. What instructions should the nurse include in the discharge teaching? a) "You will receive IV antibiotics for 3 to 6 weeks." b) "You need to limit the amount of protein and calcium in your diet." c) "You need to perform weight-bearing exercises twice a week." d) "Use your continuous passive motion machine (CPM) 2 hours each day."

"You will receive IV antibiotics for 3 to 6 weeks." Explanation: Treatment of osteomyelitis requires IV antibiotics for 3 to 6 weeks.

Traumatic rupture of the Achilles tendon occurs with the ____________ action of the gait

"pushing off"

Peak adult bone mass is achieved between the ages of ____ and ____ years in both women and men and is affected by genetic factors, nutrition, physical activity, medications, endocrine status, and general health

18 and 25

How many bones are in the body?

206

An older adult patient had a hip replacement. When should the patient begin with assisted ambulation with a walker? a. 24 hours b. 72 hours c. 1 week d. 2 to 3 weeks

24

The nurse expects that ________ of weight can be used for a patient in skeletal traction

25 pounds

After a total hip replacement, the patient is usually able to resume daily activities after ________.

3 months

After a total hip replacement, stair climbing is kept to a minimum for ________ to ________ months.

3 to 6

A client with diabetes punctured hisfoot with a broken acorn in the yard. Within a week, the client developed osteomyelitis of the foot. The client was admitted for IV antibiotic therapy. How long does the nurse anticipate the client will receive IV antibiotics? a) 3 months b) 3 to 6 weeks c) 6 months d) 7 to 10 days

3 to 6 weeks Explanation: Identification of the causative organism to initiate appropriate and ongoing antibiotic therapy for infection control. IV antibiotic therapy is administered for 3 to 6 weeks. Oral antibiotics then follow for as long as 3 months.

Zoledronic acid (Reclast) is the most potent bisphosphonate and is associated with acute kidney injury; therefore it is contraindicated in patients with creatinine clearances less than ______ mL/min or in patients with chronic kidney disease

35

The nurse caring for a postoperative hip replacement patient knows that the patient should not cross their legs at any time for ________ after surgery.

4 months

The nurse monitors the patient in a large body cast for potential physiologic cast syndrome, noting bowel sounds every ___ to ____ hours, and reports abdominal discomfort and distention, nausea, and vomiting to the primary provider.

4 to 8

Primary osteoporosis in women usually begins between the ages of ________ and ________.

45 and 55, after menopause

The patient may require a tetanus booster if the wound is dirty and if the last known booster was given more than ______ years ago.

5

The nurse is caring for a patient after arthroscopic surgery for a rotator cuff tear. The nurse informs the patient that full activity can usually resume after what period of time? a. 3 to 4 weeks b. 8 weeks c. 3 to 4 months d. 6 to 12 months

6 to 12 months

Young adults need a daily vitamin D intake of____________ IU, whereas adults 50 years and older require a daily intake of ______________ to ___________IU to ensure good bone health

600 800 to 1000

in surgery is If a wound drainage system is used, drainage of 200 to 500 mL in the first 24 hours is expected; by 48 hours postoperatively, the total drainage in ____ hours usually decreases to 30 mL or less, and the suction device is then removed; drains that remain in place for longer than 24 hours are at an increased risk for contamination, and infection may occur

8

Calcium levels

9-10.5

More than _______ of total body calcium is present in bone

98%

A client with suspected osteomalacia has a fractured tibia and fibula. What test would give a definitive diagnosis of osteomalacia? a) A bone biopsy b) Demineralization of the bone c) Increased and decreased areas of bone metabolism d) Elevated levels of alkaline phosphatase

A bone biopsy Explanation: A definitive diagnosis is obtained by bone biopsy. Radiographic studies demonstrate demineralization of the bone. A bone scan detects increased and decreased areas of bone metabolism. Alkaline phosphatase levels are detected from a blood sample.

____________________, which may be performed with or without the use of oral or intravenous (IV) contrast agents, shows a more detailed cross-sectional image of the body. It may be used to visualize and assess tumors; injury to the soft tissue, ligaments, or tendons; and severe trauma to the chest, abdomen, pelvis, head, or spinal cord. It is also used to identify the location and extent of fractures in areas that are difficult to evaluate (e.g., acetabulum) and not visible on x-ra

A computed tomography (CT) scan

Osteoporosis

A condition in which the body's bones become weak and break easily.

pathologic fracture

A fracture occurring through an area of diseased

A patient's electronic health record notes that the patient has hallux valgus. What signs and symptoms would the nurse expect this patient to manifest? A) Deviation of a great toe laterally B) Abnormal flexion of the great toe C) An exaggerated arch of the foot D) Fusion of the toe joints

Ans: A Feedback: A deformity in which the great toe deviates laterally and there is a marked prominence of the medial aspect of the first metatarsal-phalangeal joint and exostosis is referred to as hallux valgus (bunion). Hallux valgus does not result in abnormal flexion, abnormalities of the arch, or joint fusion.

A patient who has undergone a lower limb amputation is preparing to be discharged home. What outcome is necessary prior to discharge? A) Patient can demonstrate safe use of assistive devices. B) Patient has a healed, nontender, nonadherent scar. C) Patient can perform activities of daily living independently. D) Patientis free of pain.

Ans: A Feedback: A patient should be able to use assistive devices appropriately and safely prior to discharge. Scar formation will not be complete at the time of hospital discharge. It is anticipated that the patient will require some assistance with ADLs postdischarge. Pain should be well managed, but may or may not be wholly absent.

______________ are the cornerstone of Paget therapy in that they stabilize the rapid bone turnover. Their use may not suppress all Paget symptoms, but they reduce serum ALP and urinary hydroxyproline levels.

Bisphosphonates

A nurse is planning the care of an older adult patient with osteomalacia. What action should the nurse recommend in order to promote vitamin D synthesis? A) Ensuring adequate exposure to sunlight B) Eating a low-purine diet C) Performing cardiovascular exercise while avoiding weight-bearing exercises D) Taking thyroid supplements as ordered

Ans: A Feedback: Because sunlight is necessary for synthesizing vitamin D, patients should be encouraged to spend some time in the sun. A low-purine diet is not a relevant action and thyroid supplements do not directly affect bone function. Action must be taken to prevent fractures, but weight-bearing exercise within safe parameters is not necessarily contraindicated.

An older adult woman's current medication regimen includes alendronate (Fosamax). What outcome would indicate successful therapy? A) Increased bone mass B) Resolution of infection C) Relief of bone pain D) Absence of tumor spread

Ans: A Feedback: Bisphosphonates such as Fosamax increase bone mass and decrease bone loss by inhibiting osteoclast function. These drugs do not treat infection, pain, or tumors.

A nurse is teaching a patient with osteomalacia about the role of diet. What would be the best choice for breakfast for a patient with osteomalacia? A) Cereal with milk, a scrambled egg, and grapefruit B) Poached eggs with sausage and toast C) Waffles with fresh strawberries and powdered sugar D) A bagel topped with butter and jam with a side dish of grapes

Ans: A Feedback: The best meal option is the one that contains the highest dietary sources of calcium and vitamin D. The best selection among those listed is cereal with milk, and eggs, as these foods contain calcium and vitamin D in a higher quantity over the other menu options.

A nurse is caring for a patient who had a right below-the-knee amputation (BKA). The nurse recognizes the importance of implementing measures that focus on preventing flexion contracture of the hip and maintaining proper positioning. Which of the following measures will best achieve these goals? A) Encouraging the patient to turn from side to side and to assume a prone position B) Initiating ROM exercises of the hip and knee 10 to 12 weeks after the amputation C) Minimizing movement of the flexor muscles of the hip D) Encouraging the patient to sit in a chair for at least 8 hours a day

Ans: A Feedback: The nurse encourages the patient to turn from side to side and to assume a prone position, if possible, to stretch the flexor muscles and to prevent flexion contracture of the hip. Postoperative ROM exercises are started early, because contracture deformities develop rapidly. ROM exercises include hip and knee exercises for patients with BKAs. The nurse also discourages sitting for prolonged periods of time.

A patient has returned to the postsurgical unit from the PACU after an above-the-knee amputation of the right leg. Results of the nurse's initial postsurgical assessment were unremarkable but the patient has called out. The nurse enters the room and observes copious quantities of blood at the surgical site. What should be the nurse's initial action? A) Apply a tourniquet. B) Elevate the residual limb. C) Apply sterile gauze. D) Call the surgeon.

Ans: A Feedback: The nurse should apply a tourniquet in the event of postsurgical hemorrhage. Elevating the limb and applying sterile gauze are likely insufficient to stop the hemorrhage. The nurse should attempt to control the immediate bleeding before contacting the surgeon.

A nurse is caring for a patient who is 12 hours postoperative following foot surgery. The nurse assesses the presence of edema in the foot. What nursing measure will the nurse implement to control the edema? A) Elevate the foot on several pillows. B) Apply warm compresses intermittently to the surgical area. C) Administer a loop diuretic as ordered. D) Increase circulation through frequent ambulation.

Ans: A Feedback: To control the edema in the foot of a patient who experienced foot surgery, the nurse will elevate the foot on several pillows when the patient is sitting or lying. Diuretic therapy is not an appropriate intervention for edema related to inflammation. Intermittent ice packs should be applied to the surgical area during the first 24 to 48 hours after surgery to control edema and provide some pain relief. Ambulation will gradually be resumed based on the guidelines provided by the surgeon.

The nurse educator on an orthopedic trauma unit is reviewing the safe and effective use of traction with some recent nursing graduates. What principle should the educator promote? A) Knots in the rope should not be resting against pulleys. B) Weights should rest against the bed rails. C) The end of the limb in traction should be braced by the footboard of the bed. D) Skeletal traction may be removed for brief periods to facilitate the patient's independence.

Ans: A Feedback: Knots in the rope should not rest against pulleys, because this interferes with traction. Weights are used to apply the vector of force necessary to achieve effective traction and should hang freely at all times. To avoid interrupting traction, the limb in traction should not rest against anything. Skeletal traction is never interrupted.

A nurse is admitting a patient to the unit who presented with a lower extremity fracture. What signs and symptoms would suggest to the nurse that the patient may have aperoneal nerve injury? A) Numbness and burning of the foot B) Pallor to the dorsal surface of the foot C) Visible cyanosis in the toes D) Inadequate capillary refill to the toes

Ans: A Feedback: Peroneal nerve injury may result in numbness, tingling, and burning in the feet. Cyanosis, pallor, and decreased capillary refill are signs of inadequate circulation.

Bone is splintered into several fragments

Comminuted

An older adult patient experienced a fall and required treatment for a fractured hip on the orthopedic unit. Which of the following are contributory factors to the incidence of falls and fractured hips among the older adult population? Select all that apply. A) Loss of visual acuity B) Adverse medication effects C) Slowed reflexes D) Hearing loss E) Muscle weakness

Ans: A, B, C, E Feedback: Older adults are generally vulnerable to falls and have a high incidence of hip fracture. Weak quadriceps muscles, medication effects, vision loss, and slowed reflexes are among the factors that contribute to the incidence of falls. Decreased hearing is not noted to contribute to the incidence of falls.

A patient has had a brace prescribed to facilitate recovery from a knee injury. What are the potential therapeutic benefits of a brace? Select all that apply. A) Preventing additional injury B) Immobilizing prior to surgery C) Providing support D) Controlling movement E) Promoting bone remodeling

Ans: A, C, D Feedback: Braces (i.e., orthoses) are used to provide support, control movement, and prevent additional injury. They are not used to immobilize body parts or to facilitate bone remodeling.

A nurse is caring for a patient who is being treated in the hospital for a spontaneous vertebral fracture related to osteoporosis. The nurse should address the nursing diagnosis of Acute Pain Related to Fracture by implementing what intervention? A) Maintenance of high Fowler's positioning whenever possible B) Intermittent application of heat to the patient's back C) Use of a pressure-reducing mattress D) Passive range of motion exercises

Ans: B Feedback: Intermittent local heat and back rubs promote muscle relaxation following osteoporotic vertebral fractures. High Fowler's positioning is likely to exacerbate pain. The mattress must be adequately supportive, but pressure reduction is not necessarily required. Passive range of motion exercises to the back would cause pain and impair healing.

A patient has come to the clinic for a routine annual physical. The nurse practitioner notes a palpable, painless projection of bone at the patient's shoulder. The projection appears to be at the distal end of the humerus. The nurse should suspect the presence of which of the following? A) Osteomyelitis B) Osteochondroma C) Osteomalacia D) Paget's disease

Ans: B Feedback: Osteochondroma is the most common benign bone tumor. It usually occurs as a large projection of bone at the end of long bones (at the knee or shoulder). Osteomyelitis, osteomalacia, and Paget's disease do not involve the development of excess bone tissue.

what patients are not candidates for a MRI?

Because an electromagnet is used, patients with any metal implants (i.e., cochlear implants), clips, or pacemakers are not candidates for MRI

treatment for osteomyelitis

Because there is decreased penetration by medications, antibiotic therapy is longer term than with other infections; typically it continues for 3 to 6 weeks

The nurse is providing care for a patient who has had a below-the-knee amputation. The nurse enters the patient's room and finds him resting in bed with his residual limb supported on pillow. What is the nurse's most appropriate action? A) Inform the surgeon of this finding. B) Explain the risks of flexion contracture to the patient. C) Transfer the patient to a sitting position. D) Encourage the patient to perform active ROM exercises with the residual limb.

Ans: B Feedback: The residual limb should not be placed on a pillow, because a flexion contracture of the hip may result. There is no acute need to contact the patient's surgeon. Encouraging exercise or transferring the patient does not address the risk of flexion contracture.

While assessing a patient who has had knee replacement surgery, the nurse notes that the patient has developed a hematoma at the surgical site. The affected leg has a decreased pedal pulse. What would be the priority nursing diagnosis for this patient? A) Risk for Infection B) Risk for Peripheral Neurovascular Dysfunction C) Unilateral Neglect D) Disturbed Kinesthetic Sensory Perception

Ans: B Feedback: The hematoma may cause an interruption of tissue perfusion, so the most appropriate nursing diagnosis is Risk of Peripheral Neurovascular Dysfunction. There is also an associated risk for infection because of the hematoma, but impaired neurovascular function is a more acute threat. Unilateral neglect and impaired sensation are lower priorities than neurovascular status.

A patient with a right tibial fracture is being discharged home after having a cast applied. What instruction should the nurse provide in relationship to the patient's cast care? A) "Cover the cast with a blanket until the cast dries." B) "Keep your right leg elevated above heart level." C) "Use a clean object to scratch itches inside the cast." D) "A foul smell from the cast is normal after the first few days."

Ans: B Feedback: The leg should be elevated to promote venous return and prevent edema. The cast shouldn't be covered while drying because this will cause heat buildup and prevent air circulation. No foreign object should be inserted inside the cast because of the risk of cutting the skin and causing an infection. A foul smell from a cast is never normal and may indicate an infection.

A nurse is caring for a patient with a bone tumor. The nurse is providing education to help the patient reduce the risk for pathologic fractures. What should the nurse teach the patient? A) Strive to achieve maximum weight-bearing capabilities. B) Gradually strengthen the affected muscles through weight training. C) Support the affected extremity with external supports such as splints. D) Limit reliance on assistive devices in order to build strength.

Ans: C Feedback: During nursing care, the affected extremities must be supported and handled gently. External supports (splints) may be used for additional protection. Prescribed weight-bearing restrictions must be followed. Assistive devices should be used to strengthen the unaffected extremities.

A patient has come to the orthopedic clinic for a follow-up appointment 6 weeks after fracturing his ankle. Diagnostic imaging reveals that bone union is not taking place. What factor may have contributed to this complication? A) Inadequate vitamin D intake B) Bleeding at the injury site C) Inadequate immobilization D) Venous thromboembolism (VTE)

Ans: C Feedback: Inadequate fracture immobilization can delay or prevent union. A short-term vitamin D deficiency would not likely prevent bone union. VTE is a serious complication but would not be a cause of nonunion. Similarly, bleeding would not likely delay union.

A nurse is caring for a patient with Paget's disease and is reviewing the patient's most recent laboratory values. Which of the following values is most characteristic of Paget's disease? A) An elevated level of parathyroid hormone and low calcitonin levels B) A low serum alkaline phosphatase level and a low serum calcium level C) An elevated serum alkaline phosphatase level and a normal serum calcium level D) An elevated calcitonin level and low levels of parathyroid hormone

Ans: C Feedback: Patients with Paget's disease have normal blood calcium levels. Elevated serum alkaline phosphatase concentration and urinary hydroxyproline excretion reflect the increased osteoblastic activity associated with this condition. Alterations in PTH and calcitonin levels are atypical.

A patient presents at a clinic complaining of back pain that goes all the way down the back of the leg to the foot. The nurse should document the presence of what type of pain? A) Bursitis B) Radiculopathy C) Sciatica D) Tendonitis

Ans: C Feedback: Sciatica nerve pain travels down the back of the thigh to the foot of the affected leg. Bursitis is inflammation of a fluid-filled sac in a joint. Radiculopathy is disease of a nerve root. Tendonitis is inflammation of muscle tendons.

A patient is admitted to the unit in traction for a fractured proximal femur and requires traction prior to surgery. What is the most appropriate type of traction to apply to a fractured proximal femur? A) Russell's traction B) Dunlop's traction C) Buck's extension traction D) Cervical head halter

Ans: C Feedback: Buck's extension is used for fractures of the proximal femur. Russell's traction is used for lower leg fractures. Dunlop's traction is applied to the upper extremity for supracondylar fractures of the elbow and humerus. Cervical head halters are used to stabilize the neck.

A patient is complaining of pain in her casted leg. The nurse has administered analgesics and elevated the limb. Thirty minutes after administering the analgesics, the patient states the pain is unrelieved. The nurse should identify the warning signs of what complication? A) Subcutaneous emphysema B) Skin breakdown C) Compartment syndrome D) Disuse syndrome

Ans: C Feedback: Compartment syndrome may manifest as unrelenting, uncontrollable pain. This presentation of pain is not suggestive of disuse syndrome or skin breakdown. Subcutaneous emphysema is not a complication of casting.

An older adult patient sought care for the treatment of a swollen, painful knee joint. Diagnostic imaging and culturing of synovial fluid resulted in a diagnosis of septic arthritis. The nurse should prioritize which of the following aspects of care? A) Administration of oral and IV corticosteroids as ordered B) Prevention of falls and pathologic fractures C) Maintenance of adequate serum levels of vitamin D D) Intravenous administration of antibiotics

Ans: D Feedback: IV antibiotics are the major treatment modality for septic arthritis; the nurse must ensure timely administration of these drugs. Corticosteroids are not used to treat septic arthritis and vitamin D levels are not necessarily affected. Falls prevention is important, but septic arthritis does not constitute the same fracture risk as diseases with decreased bone density.

A patient who has had an amputation is being cared for by a multidisciplinary rehabilitation team. What is the primary goal of this multidisciplinary team? A) Maximize the efficiency of care B) Ensure that the patient's health care is holistic C) Facilitate the patient's adjustment to a new body image D) Promote the patient's highest possible level of function

Ans: D Feedback: The multidisciplinary rehabilitation team helps the patient achieve the highest possible level of function and participation in life activities. The team is not primarily motivated by efficiency, the need for holistic care, or the need to foster the patient's body image, despite the fact that each of these are valid goals.

A nurse is emptying an orthopedic surgery patient's closed suction drainage at the end of a shift. The nurse notes that the volume is within expected parameters but that the drainage has a foul odor. What is the nurse's best action? A) Aspirate a small amount of drainage for culturing. B) Advance the drain 1 to 1.5 cm. C) Irrigate the drain with normal saline. D) Inform the surgeon of this finding.

Ans: D Feedback: The nurse should promptly notify the surgeon of excessive or foul-smelling drainage. It would be inappropriate to advance the drain, irrigate the drain, or aspirate more drainage.

taking x-ray images after injection of contrast material into a joint

Arthrography

The nurse is preparing a client for a surgical procedure that will allow visualization of the extent of joint damage of the knee for a client with rheumatoid arthritis and also obtain a sample of synovial fluid. What procedure will the nurse prepare the client for? a) Needle aspiration b) Arthroscopy c) Arthroplasty d) Open reduction

Arthroscopy Explanation: Arthroscopic examination may be carried out to visualize the extent of joint damage as well as to obtain a sample of synovial fluid. An open reduction would be used for the treatment of a fracture. Needle aspiration will not allow visualization of the joint damage but will allow obtaining the sample of synovial fluid. Arthroplasty is the restructure of the joint surface after diagnosis is made.

A nurse is caring for an elderly female client with osteoporosis. When teaching the client, the nurse should include information about which major complication? a) Negative calcium balance b) Loss of estrogen c) Dowager's hump d) Bone fracture

Bone fracture Explanation: Bone fracture is a major complication of osteoporosis; it results when loss of calcium and phosphate increases the fragility of bones. Estrogen deficiencies result from menopause — not osteoporosis. Calcium and vitamin D supplements may be used to support normal bone metabolism, but a negative calcium balance isn't a complication of osteoporosis. Dowager's hump results from bone fractures. It develops when repeated vertebral fractures increase spinal curvature.

A 35-year-old client is visiting a rheumatology group practice. The client has experienced increasing pain and progressing inflammation of the hands and feet. The rheumatologist has prescribed NSAID use to treat the condition. What client education is most important for the nurse to address with the use of these medications? a) Activity restrictions b) Common adverse effects c) Dietary restrictions d) Loading-dose schedule

Common adverse effects Explanation: The most common adverse effects of NSAIDs are related to the GI tract: nausea, vomiting, diarrhea, and constipation. GI bleeding, which in some cases is severe, has been reported with the use of these drugs. Use of NSAIDs does not pose significant dietary or activity restrictions nor is there a loading-dose schedule.

causes for carpal syndrome

Commonly caused by repetitive hand and wrist movements, it is also associated with rheumatoid arthritis, diabetes, acromegaly, hyperthyroidism, or trauma

the most serious complication of casting and splinting?

Compartment syndrome

How is Superior mesenteric artery syndrome treated?

Cutting a window in the abdominal portion of the cast or bivalving the cast may be sufficient to prevent or relieve pressure on the duodenum.

A nurse is caring for a patient who is in skeletal traction. To prevent the complication of skin breakdown in a patient with skeletal traction, what action should be included in the plan of care? A) Apply occlusive dressings to the pin sites. B) Encourage the patient to push up with the elbows when repositioning. C) Encourage the patient to perform isometric exercises once a shift. D) Assess the pin insertion site every 8 hours.

D The pin insertion site should be assessed every 8 hours for inflammation and infection. Loose cover dressings should be applied to pin sites. The patient should be encouraged to use the overhead trapeze to shift weight for repositioning. Isometric exercises should be done 10 times an hour while awake.

physical signs of ____ include pain and tenderness at or below the area of the clot, swelling or tightness of the affected leg, possibly with pitting edema, with either warmth or cooling, and skin discoloration

DVT

Which is a flexion deformity caused by a slowly progressive contracture of the palmar fascia? a) Callus b) Hammertoe c) Hallux valgus d) Dupuytren's contracture

Dupuytren's contracture Explanation: Dupuytren's disease results in a slowly progressive contracture of the palmar fascia, called Dupuytren's contracture. A callus is a discretely thickened area of skin that has been exposed to persistent pressure or friction. A hammertoe is a flexion deformity of the interphalangeal joint, which may involve several toes. Hallux valgus is a deformity in which the great toe deviates laterally

Which of the following diagnostics are used to evaluate spinal nerve root disorders (radiculopathies)? a) Electromyogram b) Magnetic resonance imaging c) Bone scan d) Computed tomography

Electromyogram Explanation: An electromyogram and nerve conduction studies are used to evaluate spinal nerve toot disorders (radiculopathies) for patients with low back pain. A bone scan may disclose information about infections, tumors, and bone marrow abnormalities. A computed tomography scan is useful in identifying underlying problems, such as obscure soft tissue lesions adjacent to the vertebral column and problems of vertebral disks. Magnetic resonance imaging permits visualization of the nature and location of spinal pathology.

epiphysis

End of a long bone

The nurse notes that the patient's left great toe deviates laterally. This finding would be recognized as which of the following? a) Pes cavus b) Hammertoe c) Flatfoot d) Hallux valgus

Hallux valgus Explanation: Hallux valgus is commonly referred to as a bunion. Hammertoes are usually pulled upward. Pes cavus refers to a foot with an abnormally high arch and a fixed equinus deformity of the forefoot. In flatfoot, the patient demonstrates a diminished longitudinal arch of the foot.

_______ Symptom may include acute onset of dyspnea, tachycardia, confusion, and pleuritic chest pain

PE

A group of students are reviewing information about osteoporosis in preparation for a class discussion. The students demonstrate a need for additional review when they state which of the following as a risk factor? a) Hypothyroidism b) Excess caffeine intake c) Prolonged corticosteroid use d) Prolonged immobility

Hypothyroidism Explanation: Factors associated with an increased risk for osteoporosis include: family history of osteoporosis, chronic low calcium intake, excessive intake of caffeine, tobacco use, Cushing's syndrome, prolonged use of high doses of corticosteroids, prolonged periods of immobility, hyperthyroidism, hyperparathyroidism, eating disorders, malabsorption syndromes, breast cancer (especially if treated with chemotherapy that suppresses estrogen, excluding Tamoxifen, which may reduce the risk of fractures), renal or liver failure, alcoholism, lactose intolerance, and dietary deficiency of vitamin D and calcium.

Electromyography nursing interventions

If the nurse finds that the patient is taking an anticoagulant or has a skin infection, the primary provider is notified.

A female client is at risk for developing osteoporosis. Which action will reduce the client's risk? a) Taking a 300-mg calcium supplement to meet dietary guidelines b) Stopping estrogen therapy c) Living a sedentary lifestyle to reduce the incidence of injury d) Initiating weight-bearing exercise routines

Initiating weight-bearing exercise routines Explanation: Performing weight-bearing exercise increases bone health. A sedentary lifestyle increases the risk of developing osteoporosis. Estrogen is needed to promote calcium absorption. The recommended daily intake of calcium is 1,000 mg, not 300 mg.

Which should be included in the teaching plan for a patient diagnosed with plantar fasciitis? a) Plantar fasciitis presents as an acute onset of pain localized to the ball of the foot. b) Complications of plantar fasciitis include neuromuscular damage and decreased ankle range of motion. c) Management of plantar fasciitis includes stretching exercises. d) The pain of plantar fasciitis diminishes with warm water soaks.

Management of plantar fasciitis includes stretching exercises. Explanation: Management also includes wearing shoes with support and cushioning to relieve pain, orthotic devices (e.g., heel cups, arch supports), and the use of nonsteroidal anti-inflammatory drugs (NSAIDs). Plantar fasciitis, an inflammation of the foot-supporting fascia, presents as an acute onset of heel pain experienced with taking the first steps in the morning. The pain is localized to the anterior medial aspect of the heel and diminishes with gentle stretching of the foot and Achilles tendon. Unresolved plantar fasciitis may progress to fascial tears at the heel and eventual development of heel spurs.

Localized rapid bone turnover, most commonly affecting the skull, femur, tibia, pelvic bones, and vertebrae, is characterized by which of the following bone disorders? a) Osteomyelitis b) Ganglion c) Osteomalacia d) Paget's disease

Paget's disease Explanation: Paget's disease results in bone that is highly vascularized and structurally weak, predisposing to pathologic fractures. Osteomalacia is a metabolic bone disease characterized by inadequate mineralization of bone. A ganglion is a collection of neurological gelatinous material. Osteomyelitis is an infection of bone that comes from the extension of a soft tissue infection, direct bone contamination, or hematogenous spread.

Balanced suspension

Provides the countertraction so that the pulling force of the traction is not altered when the bed or patient is moved

The nurse is caring for a patient who had a total hip replacement. What lethal postoperative complication should the nurse closely monitor for? a. Atelectasis b. Hypovolemia c. Pulmonary embolism d. Urinary tract infection

Pulmonary embolism

late stages of compartment syndrome

Pulselessness, paresthesia, and complete paralysis are found in the late stages of compartment syndrome

A patient arrives in the emergency department with a suspected bone fracture of the right arm. How does the nurse expect the patient to describe the pain? a. A dull, deep, boring ache b. Sharp and piercing c. Similar to "muscle cramps" d. Sore and aching

Sharp and piercing

A male client is to have an amputation. He is acutely ill and diagnosed with a gangrenous limb and related fever, disorientation, and electrolyte imbalances. Which of the following would be most important for the nurse to monitor in this client? a) Signs of nausea and vomiting b) Signs of sepsis c) Occurrence of allergic reactions d) Reduced urine output

Signs of sepsis Explanation: If the client is acutely ill with a gangrenous limb, related fever, disorientation, and electrolyte imbalances, the nurse should monitor for signs of sepsis and circulation in the limb for any changes such as severe pain, color changes, and lack of peripheral pulses. It is crucial for the nurse to inform the physician about the problems as they occur or else the surgery may become an emergency. Monitoring for signs of nausea and vomiting, occurrence of allergic reactions, and reduced urine output, although necessary, is not as crucial for the client.

More than 50% of bone infections are caused by ______________ and increasingly of the variety that is methicillin resistant

Staphylococcus aureus

Septic (Infectious) Arthritis

Swollen, painful, red, monoarticular arthritis with migratory involvement and assymetrical pattern of involvement

The nurse is educating a patient with lower back pain on proper lifting techniques. The nurse would document what behavior as evidence the education was effective? a) The patient used a narrow base of support. b) The patient placed the load close to the body. c) The patient reached over head with arms fully extended. d) The patient bent at the hips and tightened the abdominal muscles.

The patient placed the load close to the body. Explanation: Instructions for the patient with low back pain should include that when lifting, the patient should avoid overreaching. The patient should also keep the load close to the body, bend the knees and tighten the abdominal muscles, use a wide base of support, and use a back brace to protect the back. Bending at the hips increases the strain on the back muscles when lifting.

A client with low back pain is being seen in the clinic. In planning care, which teaching point should the nurse include? a) Sleep on the stomach to alleviate pressure on the back. b) Use the large muscles of the leg when lifting items. c) Avoid twisting and flexion activities. d) A soft mattress is most supportive by conforming to the body.

Use the large muscles of the leg when lifting items. Explanation: The large muscles of the leg should be used when lifting.

_______ is one of the most common and dangerous of all complications occurring in the postoperative orthopedic patient.

VTE

A patient is diagnosed with osteomyelitis of the right leg. What signs and symptoms does the nurse recognize that are associated with this diagnosis? (Select all that apply.) a. Pain in the right leg b. Erythema of the right leg c. Fever d. Leukopenia e. Purulent drainage

a. Pain in the right leg b. Erythema of the right leg c. Fever

Common side effects of calcium supplements

abdominal distention and constipation.

Which of the following are clinical manifestations of impingement syndrome? Select all that apply. a) Limited movement b) Atrophy c) Pain d) Shoulder tenderness e) Muscle spasms

• Pain • Shoulder tenderness • Limited movement • Muscle spasms • Atrophy Explanation: The patient experiences pain, shoulder tenderness, limited movement, muscle spasms, and atrophy. The process may progress to a rotator cuff tear.

The nurse is caring for the client with chronic osteomyelitis of the jaw with a draining wound. Which client goal is a priority for the client? a) The client will maintain adequate nutritional intake. b) The client will maintain effective airway clearance. c) The client will experience a tolerable level of pain. d) The client will demonstrate wound care. e) The client will remain free from injury.

• The client will experience a tolerable level of pain. • The client will demonstrate wound care. • The client will maintain adequate nutritional intake. Explanation: Pain is a priority problem for the client with osteomyelitis, and it can interfere with mobility of joint. In this situation, the client's jaw is the site of infection. Pain in this location can interfere with nutritional intake of the individual. Chronic osteomyelitis presents with a nonhealing ulcer over the infected bone with a connecting sinus that will intermittently and spontaneously drain pus. The client will need to be able to provide wound care in the home setting. Remaining free from injury and maintaining an effective airway clearance are not priority goals for the client.

Ligaments

Connect bone to bone

abnormal shortening of muscle or fibrosis of joint structures

contracture

osteopenia

abnormal reduction of bone mass

atrophy

decrease in muscle size

Bisphosphonates Adverse effects

esophageal or gastric ulcers, osteonecrosis of the jaw, and atypical femur fractures

Plantar Fasciitis

- an inflammation of the foot-supporting fascia - presents as an acute onset of heel pain experienced with the first steps in the morning.

A patient diagnosed with carpal tunnel syndrome (CTS) asks the nurse about numbness in the fingers and pain in the wrist. In responding to the patient, how would the nurse best describe CTS? a. "CTS is a neuropathy that is characterized by bursitis and tendonitis" b. "CTS is a neuropathy that is characterized by flexion contracture of the fourth and fifth fingers" c. "CTS is a neuropathy that is characterized by compression of the median nerve at the wrist" d. "CTS is a neuropathy that is characterized by pannus formation in the shoulder"

"CTS is a neuropathy that is characterized by compression of the median nerve at the wrist"

If pressure necrosis occurs, the patient typically reports a very painful ______________ and tightness under the cast. The cast may feel warmer in the affected area, suggesting underlying tissue erythema.

"hot spot"

Nursing Management of the Patient With an Immobilized Lower Extremity

-leg must be supported on pillows to the level of the heart to control swelling - Cold therapy or ice packs (first 1 to 2 says)

Bursitis

Bursae are fluid-filled sacs that prevent friction between joint structures during joint activity and are painful when inflamed * when these sacs become inflamed

Compare the advantages of a fiberglass cast to those of a plaster cast.

A fiberglass cast is light in weight and water resistant. It is more durable than plaster and water resistant

Joint replacement

A surgical procedure in which a mechanical device, designed to act as a joint, is used to replace a diseased joint; most common joints: hip, knee, shoulder, finger; accurate fitting is essential; excellent pain relief; infection is a post-op concern

Calcitriol

active form of vitamin D

A nurse is reviewing a patient's activities of daily living prior to discharge from total hip replacement. The nurse should identify what activity as posing a potential risk for hip dislocation? A) Straining during a bowel movement B) Bending down to put on socks C) Lifting items above shoulder level D) Transferring from a sitting to standing position

Ans: B Feedback: Bending to put on socks or shoes can cause hip dislocation. None of the other listed actions poses a serious threat to the integrity of the new hip.

________ is the most common fracture of the distal radius.

Colles' fracture

Which assessment findings would the nurse expect in the client with osteomalacia? a) Column A b) Column B c) Column C d) Column D

Column B Explanation: Osteomalacia is characterized by decreased serum calcium and phosphorus and elevated alkaline phosphatase levels

Name three major complications of an extremity that is casted, braced, or splinted.

Compartment syndrome, pressure ulcers, and disuse syndrome

cortisol function musculoskeletal system

Cortisol inhibits bone formation and decreases calcium absorption in the intestine

Meniscal Injuries treatment

Initial conservative treatment includes immobilization of the knee, cryotherapy, the use of crutches for support, anti-inflammatory agents, analgesic agents

lamellae

Layers of bone matrix

______________ ("joint mice") may occur in a joint space as a result of articular cartilage wear and bone erosion. These fragments can interfere with joint movement ("locking the joint").

Loose bodies

meniscus function

Lubrication and nourishment of the knee joint, weight distribution, and assistance with joint biomechanics

hyaline cartilage

Most common type of cartilage; it is found on the ends of long bones, ribs, and nose

A family member is caring for an older adult client with osteomalacia in the home. When the home health nurse comes to evaluate the client, what should be a focus point of the visit? a) Making sure the client has adequate financial resources b) Observing for safety hazards that could be a fall risk c) Ensuring that the client is eating enough d) Making sure the client is receiving a daily bath

Observing for safety hazards that could be a fall risk Explanation: Clients with osteomalacia exhibit a waddling type of gait, putting them at risk for falls and fractures. Safety would be the priority in this circumstance such as scatter rugs, loose boards, and stairs. Older adult clients do not require a daily bath, and it may dry the skin. Nutrition is a necessity to question but the priority would be safety. Whether the client has adequate financial resources would be referred to social service.

_________ is the most common benign bone tumor. It usually occurs as a large projection of bone at the end of long bones (at the knee or shoulder), developing during growth. It then becomes a static bony mass.

Osteochondroma

parathyroid hormone and calcitonin

Parathyroid hormone acts to INCREASE blood calcium levels, while calcitonin acts to DECREASE blood calcium levels.

Primary osteoporosis

Patients have a low bone mass but have normal serum calcium, phosphorous, and PTH levels.

The nurse is caring for the client following removal of a Morton's neuroma. Which of the following nursing interventions would be inappropriate? a) Assess the surgical dressing. b) Assist the client with incentive spirometry. c) Elevate the foot on two pillows. d) Perform neurovascular assessment of the hand.

Perform neurovascular assessment of the hand. Explanation: Morton's neuroma is a foot problem characterized by swelling of the median plantar nerve.

What potential immobility-related complications may develop when a patient is in skeletal traction?

Pressure ulcers, atelectasis, pneumonia, constipation, anorexia, urinary stasisand infection, and venous thromboemboli with PE or DVT

Which of the following is the only selective estrogen receptor modulator approved for osteoporosis in post menopausal women? a) Forteo b) Fosamax c) Raloxifene d) Denosumab

Raloxifene Explanation: Raloxifene is the only selective estrogen receptor modulator (SERM) approved for osteoporosis in post menopausal women as it does not increase the risk of breast or uterine cancer, but it does come with an increased risk of thromboembolism. Fosamax is a bisphosphonate. Forteo is a subcutaneously administered medication that is given one daily for the treatment of osteoporosis. Denosumab has recently been approved for treatment of postmenopausal women with osteoporosis who are at risk for fractures.

Estrogen Agonist/Antagonist

Raloxifene (Evista) -Promotes estrogenic effects on bone, preserving BMD, with concomitant antiestrogenic effects on the uterus and breasts

Which is a risk-lowering strategy for osteoporosis? a) Increased age b) Smoking cessation c) Diet low in calcium and vitamin D d) Low initial bone mass

Smoking cessation Explanation: Risk-lowering strategies include increased dietary calcium and vitamin D intake, smoking cessation, alcohol and caffeine consumption in moderation, and outdoor activity. Individual risk factors include low initial bone mass and increased age. A lifestyle risk factor is a diet low in calcium and vitamin D.

Balloon sign

TESTS FOR MAJOR EFFUSION 1. Compress subpatellar pouch with one hand 2. Rest other hand on sides of patella 3. BALLOONING out of SIDES of PATELLA = LARGE EFFUSION

A nurse is performing discharge teaching for an elderly client with osteoporosis. Which instruction about taking a calcium supplement should the nurse include? a) Take weekly on the same day and at the same time. b) Remain in an upright position 30 minutes after taking the supplement. c) Take the supplement on an empty stomach with a full glass of water. d) Take the supplement with meals or with orange juice.

Take the supplement with meals or with orange juice. Explanation: Calcium supplements, such as Caltrate or Citracal, are over-the-counter medications. They should be taken with meals or with a beverage high in vitamin C

paresthesia

abnormal sensation of numbness and tingling without objective cause

contracture

abnormal shortening of muscle, joint, or both

A patient had a total left hip arthroplasty. What clinical manifestation would indicate to the nurse that the prosthesis is dislocated? a. The left leg is internally rotated b. The leg length is the same as the right leg c. The patient has discomfort when moving in the bed d. Diminished peripheral pulses on the affected extremity

The left leg is internally rotated

The nurse feels a grating sensation in a patient's extremity. What is this sensation caused by? How would the nurse document this sensation?

The sensation is caused by the rubbing of bone fragments against each other;the nurse would document it as crepitus.

physiologic components of cast syndrome

With decreased physical activity, gastrointestinal motility decreases, and intestinal gases accumulate. The patient exhibits abdominal distention and discomfort, nausea, and vomiting, leading to food aversion, poor intake, and weight loss. This may eventually lead to increased abdominal pressure, and ileus.

myelogram

X-ray record of the spinal cord

The primary deficit in osteomalacia is ________, which promotes calcium absorption from the gastrointestinal tract.

a deficiency in activated vitamin D (calcitriol)

Electromyography

a diagnostic test that measures the electrical activity within muscle fibers in response to nerve stimulation

strain

a musculotendinous stress injury

effusion

excess fluid in joint

Spinal Movements

flexion, extension, lateral flexion, rotation

Superior mesenteric artery syndrome

occurs when the transverse portion of the duodenum is entrapped between the SMA and aorta, causing symptoms of partial intestinal obstruction.

Alendronate (Fosamax)

- Prevention and treatment of osteoporosis in women who are postmenopausal - Administer PO, either daily or weekly - Advise patient to take in AM on empty stomach with 250 mL of water while sitting upright and to remain upright for at least 30 minutes

patient having an arthrogram nursing interventions

- pt may feel some discomfort or tingling during the procedure. - After the arthrogram, a compression elastic bandage may be applied if prescribed - joint is usually rested for 12 hours. - Strenuous activity should be avoided until approved by the primary provider. - comfort measures (e.g., mild analgesia, ice) - explain patient that it is normal to experience clicking or crackling in the joint for 24 to 48 hours after the procedure until the contrast agent or air is absorbed

Recommended calcium intake

1,000-1,200 mg/day

The most common benign bone tumor is ________.

osteochondroma

A rehabilitation nurse is working with a patient who has had a below-the-knee amputation. The nurse knows the importance of the patient's active participation in self-care. In order to determine the patient's ability to be an active participant in self-care, the nurse should prioritize assessment of what variable? A) The patient's attitude B) The patient's learning style C) The patient's nutritional status D) The patient's presurgical level of function

Ans: A Feedback: Amputation of an extremity affects the patient's ability to provide adequate self-care. The patient is encouraged to be an active participant in self-care. The patient and the nurse need to maintain positive attitudes and to minimize fatigue and frustration during the learning process. Balanced nutrition and the patient's learning style are important variables in the rehabilitation process but the patient's attitude is among the most salient variables. The patient's presurgical level of function may or may not affect participation in rehabilitation.

A nurse is providing discharge teaching for a patient who underwent foot surgery. The nurse is collaborating with the occupational therapist and discussing the use of assistive devices. On what variables does the choice of assistive devices primarily depend? A) Patient's general condition, balance, and weight-bearing prescription B) Patient's general condition, strength, and gender C) Patient's motivation, age, and weight-bearing prescription D) Patient's occupation, motivation, and age

Ans: A Feedback: Assistive devices (e.g., crutches, walker) may be needed. The choice of the devices depends on the patient's general condition and balance, and on the weight-bearing prescription. The patient's strength, motivation, and weight restrictions are not what the choice of assistive devices is based on.

An elderly female with osteoporosis has been hospitalized. Prior to discharge, when teaching the patient, the nurse should include information about which major complication of osteoporosis? A) Bone fracture B) Loss of estrogen C) Negative calcium balance D) Dowager's hump

Ans: A Feedback: Bone fracture is a major complication of osteoporosis that results when loss of calcium and phosphate increases the fragility of bones. Estrogen deficiencies result from menopause, not osteoporosis. Calcium and vitamin D supplements may be used to support normal bone metabolism, but a negative calcium balance is not a complication of osteoporosis. Dowager's hump results from bone fractures. It develops when repeated vertebral fractures increase spinal curvature.

The health care team is caring for a patient with osteomalacia. It has been determined that the osteomalacia is caused by malabsorption. What is the usual treatment for osteomalacia caused by malabsorption? A) Supplemental calcium and increased doses of vitamin D B) Exogenous parathyroid hormone and multivitamins C) Colony-stimulating factors and calcitonin D) Supplemental potassium and pancreatic enzymes

Ans: A Feedback: If osteomalacia is caused by malabsorption, increased doses of vitamin D, along with supplemental calcium, are usually prescribed.

A patient is undergoing diagnostic testing for osteomalacia. Which of the following laboratory results is most suggestive of this diagnosis? A) High chloride, calcium, and magnesium B) High parathyroid and calcitonin levels C) Low serum calcium and magnesium levels D) Low serum calcium and low phosphorus level

Ans: D Feedback: Laboratory studies will reveal a low serum calcium and low phosphorus level.

A nurse is writing a care plan for a patient admitted to the emergency department (ED) with an open fracture. The nurse will assign priority to what nursing diagnosis for a patient with an open fracture of the radius? A) Risk for Infection B) Risk for Ineffective Role Performance C) Risk for Perioperative Positioning Injury D) Risk for Powerlessness

Ans: A Feedback: The patient has a significant risk for osteomyelitis and tetanus due to the fact that the fracture is open. Powerlessness and ineffective role performance are psychosocial diagnoses that may or may not apply, and which would be superseded by immediate physiologic threats such as infection. Surgical positioning injury is not plausible, since surgery is not likely indicated.

A nurse is planning the care of a patient who will require a prolonged course of skeletal traction. When planning this patient's care, the nurse should prioritize interventions related to which of the following risk nursing diagnoses? A) Risk for Impaired Skin Integrity B) Risk for Falls C) Risk for Imbalanced Fluid Volume D) Risk for Aspiration

Ans: A Feedback: Impaired skin integrity is a high-probability risk in patients receiving traction. Falls are not a threat, due to the patient's immobility. There are not normally high risks of fluid imbalance or aspiration associated with traction.

A nurse is planning the care of a patient who has undergone orthopedic surgery. What main goal should guide the nurse's choice of interventions? A) Improving the patient's level of function B) Helping the patient come to terms with limitations C) Administering medications safely D) Improving the patient's adherence to treatment

Ans: A Feedback: Improving function is the overarching goal after orthopedic surgery. Some patients may need to come to terms with limitations, but this is not true of every patient. Safe medication administration is imperative, but this is not a goal that guides other aspects of care. Similarly, adherence to treatment is important, but this is motivated by the need to improve functional status.

A nurse is caring for a patient who is being assessed following complaints of severe and persistent low back pain. The patient is scheduled for diagnostic testing in the morning. Which of the following are appropriate diagnostic tests for assessing low back pain? that apply. A) Computed tomography (CT) B) Angiography C) Magnetic resonance imaging (MRI) D) Ultrasound E) X-ray

Ans: A, C, D, E Feedback: A variety of diagnostic tests can be used to address lower back pain, including CT, MRI, ultrasound, and X-rays. Angiography is not related to the etiology of back pain.

Which of the following patients should the nurse recognize as being at the highest risk for the development of osteomyelitis? A) A middle-age adult who takes ibuprofen daily for rheumatoid arthritis B) An elderly patient with an infected pressure ulcer in the sacral area C) A 17-year-old football player who had orthopedic surgery 6 weeks prior D) An infant diagnosed with jaundice

Ans: B Feedback: Patients who are at high risk of osteomyelitis include those who are poorly nourished, elderly, and obese. The elderly patient with an infected sacral pressure ulcer is at the greatest risk for the development of osteomyelitis, as this patient has two risk factors: age and the presence of a soft-tissue infection that has the potential to extend into the bone. The patient with rheumatoid arthritis has one risk factor and the infant with jaundice has no identifiable risk factors. The patient 6 weeks postsurgery is beyond the usual window of time for the development of a postoperative surgical wound infection.

A patient has recently been admitted to the orthopedic unit following total hip arthroplasty. The patient has a closed suction device in place and the nurse has determined that there were 320 mL of output in the first 24 hours. How should the nurse best respond to this assessment finding? A) Inform the primary care provider promptly. B) Document this as an expected assessment finding. C) Limit the patient's fluid intake to 2 liters for the next 24 hours. D) Administer a loop diuretic as ordered.

Ans: B Feedback: Drainage of 200 to 500 mL in the first 24 hours is expected. Consequently, the nurse does not need to inform the physician. Fluid restriction and medication administration are not indicated.

A patient has suffered a muscle strain and is complaining of pain that she rates at 6 on a 10-point scale. The nurse should recommend what action? A) Taking an opioid analgesic as ordered B) Applying a cold pack to the injured site C) Performing passive ROM exercises D) Applying a heating pad to the affected muscle

Ans: B Feedback: Most pain can be relieved by elevating the involved part, applying cold packs, and administering analgesics as prescribed. Heat may exacerbate the pain by increasing blood circulation, and ROM exercises would likely be painful. Analgesia is likely necessary, but NSAIDs would be more appropriate than opioids.

A nurse in a busy emergency department provides care for many patients who present with contusions, strains, or sprains. Treatment modalities that are common to all of these musculoskeletal injuries include which of the following? Select all that apply. A) Massage B) Applying ice C) Compression dressings D) Resting the affected extremity E) Corticosteroids F) Elevating the injured limb

Ans: B, C, D, F Feedback: Treatment of contusions, strains, and sprains consists of resting and elevating the affected part, applying cold, and using a compression bandage. Massage and corticosteroids are not used to treat these injuries.

A nurse is reviewing the pathophysiology that may underlie a patient's decreased bone density. What hormone should the nurse identify as inhibiting bone resorption and promoting bone formation? A) Estrogen B) Parathyroid hormone (PTH) C) Calcitonin D) Progesterone

Ans: C Feedback: Calcitonin inhibits bone resorption and promotes bone formation, estrogen inhibits bone breakdown, and parathyroid increases bone resorption. Estrogen, which inhibits bone breakdown, decreases with aging. Parathyroid hormone (PTH) increases with aging, increasing bone turnover and resorption. Progesterone is the major naturally occurring human progestogen and plays a role in the female menstrual cycle.

A nurse is discussing conservative management of tendonitis with a patient. Which of the following may be an effective approach to managing tendonitis? A) Weight reduction B) Use of oral opioid analgesics C) Intermittent application of ice and heat D) Passive range of motion exercises

Ans: C Feedback: Conservative management of tendonitis includes rest of the extremity, intermittent ice and heat to the joint, and NSAIDs. Weight reduction may prevent future injuries but will not relieve existing tendonitis. Range-of-motion exercises may exacerbate pain. Opioids would not be considered a conservative treatment measure.

A nursing educator is reviewing the risk factors for osteoporosis with a group of recent graduates. What risk factor of the following should the educator describe? A) Recurrent infections and prolonged use of NSAIDs B) High alcohol intake and low body mass index C) Small frame, female gender, and Caucasian ethnicity D) Male gender, diabetes, and high protein intake

Ans: C Feedback: Small-framed, nonobese Caucasian women are at greatest risk for osteoporosis. Diabetes, high protein intake, alcohol use, and infections are not among the most salient risk factors for osteoporosis.

___________ is a metabolic bone disease characterized by inadequate mineralization of bone. As a result, the skeleton softens and weakens, causing pain, tenderness to touch, bowing of the bones, and pathologic fractures.

osteomalacia

A patient is being treated for a fractured hip and the nurse is aware of the need to implement interventions to prevent muscle wasting and other complications of immobility. What intervention best addresses the patient's need for exercise? A) Performing gentle leg lifts with both legs B) Performing massage to stimulate circulation C) Encouraging frequent use of the overbed trapeze D) Encouraging the patient to log roll side to side once per hour

Ans: C Feedback: The patient is encouraged to exercise as much as possible by means of the overbed trapeze. This device helps strengthen the arms and shoulders in preparation for protected ambulation. Independent logrolling may result in injury due to the location of the fracture. Leg lifts would be contraindicated for the same reason. Massage by the nurse is not a substitute for exercise.

A patient is admitted to the orthopedic unit with a fractured femur after a motorcycle accident. The patient has been placed in traction until his femur can be rodded in surgery. For what early complications should the nurse monitor this patient? Select all that apply. A) Systemic infection B) Complex regional pain syndrome C) Deep vein thrombosis D) Compartment syndrome E) Fat embolism

Ans: C, D, E Feedback: Early complications include shock, fat embolism, compartment syndrome, and venous thromboemboli (deep vein thrombosis [DVT], pulmonary embolism [PE]). Infection and CRPS are later complications of fractures.

This is a painful pinching of soft tissues in your shoulder. It happens when these tissues rub and press against a part of your shoulder blade called the "acromion." This can irritate your rotator cuff tendons, and also a soft sac called the "subacromial bursa."

Impingement Syndrome

A nurse is reviewing the care of a patient who has a long history of lower back pain that has not responded to conservative treatment measures. The nurse should anticipate the administration of what drug? A) Calcitonin B) Prednisone C) Aspirin D) Cyclobenzaprine

Ans: D Feedback: Short-term prescription muscle relaxants (e.g., cyclobenzaprine [Flexeril]) are effective in relieving acute low back pain. ASA is not normally used for pain control, due to its antiplatelet action and associated risk for bleeding. Calcitonin and corticosteroids are not used in the treatment of lower back pain.

an infection of the bone that results in inflammation, necrosis, and formation of new bone.

Osteomyelitis

Of the following, which is not a risk factor for osteoporosis? a) Being male b) Being postmenopausal c) Small-framed, thin White or Asian women d) Family history

Being male Explanation: Being male is not considered a risk factor. The following are some of the risk factors for osteoporosis: being a small-framed, thin White or Asian women; being postmenopausal; family history; inactivity; chronic low calcium intake; and excessive caffeine or tobacco use.

What food can the nurse suggest to the client at risk for osteoporosis? a) Broccoli b) Bananas c) Carrots d) Chicken

Broccoli Explanation: Calcium is important for the prevention of osteoporosis. Broccoli is high in calcium.

an entrapment neuropathy that occurs when the median nerve at the wrist is compressed by a thickened flexor tendon sheath, skeletal encroachment, edema, or a soft tissue mass.

Carpal tunnel syndrome

Meniscal Injuries surgical treatment

Damaged cartilage is surgically removed (meniscectomy) arthroscopically

The nurse is planning an education program for women of childbearing years. The nurse recognizes that primary prevention of osteoporosis includes: a) Ensuring adequate calcium and vitamin D intake b) Engaging in non-weight-bearing exercises daily c) Having a DXA beginning at age 35 years d) Undergoing assessment of serum calcium levels every year

Ensuring adequate calcium and vitamin D intake Explanation: Nutritional intake of calcium and vitamin D are essential for the prevention of osteoporosis.

A fracture that occurs through the epiphysis

Epiphyseal

A patient in pelvic traction needs circulatory status assessed. How should the nurse assess for a positive Homans' sign? a. Have the patient extend both hands while the nurse compares the volume of both radial pulses. b. Have the patient extend each leg and dorsiflex each foot to determine if pain or tenderness is present in the lower leg. c. Have the patient plantar flex both feet while the nurse performs the blanch test on all of the patient's toes. d. Have the patient squeeze the nurse's hands with their hands to evaluate any difference in strength.

Have the patient extend each leg and dorsiflex each foot to determine if pain or tenderness is present in the lower leg.

Assessment of a client reveals signs and symptoms of Paget's disease. Which of the following would be most likely? a) Skull narrowing b) Waddling gait c) Lordosis d) Long bone bowing

Long bone bowing Explanation: Some clients with Paget's disease are asymptomatic with only some mild skeletal deformity. Other clients have marked skeletal deformities which may include enlargement of the skull, bowing of the long bones, and kyphosis. Waddling gait is associated with osteomalacia.

Which area of the spinal column is subject to the greatest mechanical stress and degenerative changes? a) Lower lumbar b) Cervical c) Thoracic d) Upper lumbar

Lower lumbar Explanation: The lower lumbar disks, L4 to L5 and L5 to S1, are subject to the greatest mechanical stress and greatest degenerative changes.

CONTRACTION OF MUSCLE CAUSES __________

MOVEMENT

Which of the following presents with an onset of heel pain with the first steps of the morning? a) Plantar fasciitis b) Morton's neuroma c) Ganglion d) Hallux valgus

Plantar fasciitis Explanation: Plantar fasciitis, an inflammation of the foot-supporting fascia, present as an acute onset of heal pain experienced with the first steps in the morning. Hallux valgus (commonly called a bunion) is a deformity in which the great toe deviates laterally. Morton's neuroma is a swelling of the third (lateral) branch of the median plantar nerve. A ganglion, a collection of gelatinous material near the tendon sheaths and joints, appears as a round, firm compressible cystic swelling, usually on the dorsum of the wrist.

The nurse recognizes that goal of treatment for metastatic bone cancer is to: a) Promote pain relief and quality of life b) Diagnose the extent of bone damage c) Reconstruct the bone with a prosthesis d) Cure the diseased bone and cartilage

Promote pain relief and quality of life Explanation: Treatment of metastatic bone cancer is palliative.

The nurse is educating the patient with low back pain about the proper way to lift objects. What muscle should the nurse encourage the patient to maximize? a. Gastrocnemius b. Latissimus dorsi c. Quadriceps d. Rectus abdominis

Quadriceps

tendonitis treatment

RICE (Rest Ice Compression Elevation) NSAIDS ( Ibuprofen, Naproxen)

A nursing goal for a patient with skeletal traction is to avoid infection and the development of ________ at the site of pin insertion

osteomyelitis

A patient with chronic osteomyelitis has undergone 6 weeks of antibiotic therapy. There is no improvement in the wound appearance. What action would the nurse anticipate to promote healing? a) Wound irrigation b) Surgical debridement c) Wound packing d) Vitamin supplements

Surgical debridement Explanation: In chronic osteomyelitis, surgical debridement is used when the wound fails to respond to antibiotic therapy. Wound packing, vitamin supplements, and wound irrigation are not the standard of care when treating chronic osteomyelitis.

Morton's neuroma is exhibited by which of the following clinical manifestations? a) Swelling of the third (lateral) branch of the median plantar nerve b) Inflammation of the foot-supporting fascia c) High arm and a fixed equinus deformity d) Longitudinal arch of the foot is diminished

Swelling of the third (lateral) branch of the median plantar nerve Explanation: Morton's neuroma is swelling of the third branch of the median plantar nerve. Pes cavus refers to a foot with an abnormally high arch and a fixed equinus deformity of the forefoot. Flatfoot is a common disorder in which the longitudinal arch of the foot is diminished. Plantar fasciitis is an inflammation of the foot-supporting fascia.

Bone formation is enhanced by the stress of weight and muscle activity. When immobilized by casts, general inactivity, paralysis, or other disability, the bone is resorbed faster than it is formed, and osteoporosis results

TRUE

The nurse is educating a patient with lower back pain on proper lifting techniques. The nurse would document what behavior as evidence the education was effective? a) The patient used a narrow base of support. b) The patient bent at the hips and tightened the abdominal muscles. c) The patient placed the load close to the body. d) The patient reached over head with arms fully extended.

The patient placed the load close to the body. Explanation: Instructions for the patient with low back pain should include that when lifting, the patient should avoid overreaching. The patient should also keep the load close to the body, bend the knees and tighten the abdominal muscles, use a wide base of support, and use a back brace to protect the back. Bending at the hips increases the strain on the back muscles when lifting.

thyroid hormone function musculoskeletal system

Thyroid hormone (T3) is required for skeletal development during childhood and T3 regulates bone turnover and mineralisation in adults

A male patient with a musculoskeletal injury is instructed to alter his diet. The objective of this diet alteration is to facilitate the absorption of calcium from food and supplements. Considering the food intake objective, which of the following food items should the nurse encourage the patient to include in the diet? a) Vitamin D-fortified milk b) Bananas c) Red meat d) Green vegetables

Vitamin D-fortified milk Explanation: The nurse should advise the patient to include dietary sources of vitamin D, such as fatty fish, vitamin D-fortified milk, and cereals. These foods protect against bone loss and decrease the risk of fracture by facilitating the absorption of calcium from food and supplements. Red meat, bananas, and green vegetables do not facilitate calcium absorption from food and supplements.

a small, local, involuntary muscle contraction and relaxation which may be visible under the skin.

fasciculation of muscle

Joint pain

felt around or in the joint and typically worsens with movement

Cruciate Ligament injury treatment

- Immediate postinjury management includes PRICE and stabilization of the joint until it is evaluated for a fracture - early treatment involves application of a brace and physical therapy

open fractures grades

- Type I is a clean wound less than 1 cm long. - Type II is a larger wound without extensive soft tissue damage or avulsions. - Type III is highly contaminated and has extensive soft tissue damage. (MOST SEVERE)

If a dislocation or subluxation is not reduced immediately, ____________ may develop.

- avascular necrosis (AVN) -AVN of bone is caused by ischemia, which leads to necrosis or death of the bone cells.

manifestations of rotator cuff tear?

- patient complains of aching pain that is typically insidious in nature (unless related to an acute injury) and worsens with use. - f tenderness to palpation and difficulty sleeping on affected side - will exhibit decreased ROM and decreased strength.

What immediate nursing and medical management techniques are used for an open fracture?

- wound is covered with a sterile dressing to prevent contamination of deeper tissues. - No attempt is made to reduce the fracture,even if one of the bone fragments is protruding through the wound. - Splints are applied for immobilization

avascular necrosis (AVN)

A disease caused by the temporary or permanent loss of blood supply to bones; bone lacking blood can collapse and die

The surgical nurse is admitting a patient from postanesthetic recovery following the patient's below-the-knee amputation. The nurse recognizes the patient's high risk for postoperative hemorrhage and should keep which of the following at the bedside? A) A tourniquet B) A syringe preloaded with vitamin K C) A unit of packed red blood cells, placed on ice D) A dose of protamine sulfate

Ans: A Feedback: Immediate postoperative bleeding may develop slowly or may take the form of massive hemorrhage resulting from a loosened suture. A large tourniquet should be in plain sight at the patient's bedside so that, if severe bleeding occurs, it can be applied to the residual limb to control the hemorrhage. PRBCs cannot be kept at the bedside. Vitamin K and protamine sulfate are antidotes to warfarin and heparin, but are not administered to treat active postsurgical bleeding.

The patient scheduled for a Syme amputation is concerned about the ability to eventually stand on the amputated extremity. How should the nurse best respond to the patient's concern? A) "You will eventually be able to withstand full weight-bearing after the amputation." B) "You will have minimal weight-bearing on this extremity but you'll be taught how to use an assistive device." C) "You likely will not be able to use this extremity but you will receive teaching on use of a wheelchair." D) "You will be fitted for a prosthesis which may or may not allow you to walk."

Ans: A Feedback: Syme amputation (modified ankle disarticulation amputation) is performed most frequently for extensive foot trauma and produces a painless, durable extremity end that can withstand full weight-bearing. Therefore, each of the other teaching statements is incorrect.

An emergency department nurse is assessing a 17-year-old soccer player who presented with a knee injury. The patient's description of the injury indicates that his knee was struck medially while his foot was on the ground. The nurse knows that the patient likely has experienced what injury? A) Lateral collateral ligament injury B) Medial collateral ligament injury C) Anterior cruciate ligament injury D) Posterior cruciate ligament injury

Ans: A Feedback: When the knee is struck medially, damage may occur to the lateral collateral ligament. If the knee is struck laterally, damage may occur to the medial collateral ligament. The ACL and PCL are not typically injured in this way.

A nurse is planning the care of an older adult patient who will soon be discharged home after treatment for a fractured hip. In an effort to prevent future fractures, the nurse should encourage which of the following? Select all that apply. A) Regular bone density testing B) A high-calcium diet C) Use of falls prevention precautions D) Use of corticosteroids as ordered E) Weight-bearing exercise

Ans: A, B, C, E Feedback: Health promotion measures after an older adult's hip fracture include weight-bearing exercise, promotion of a healthy diet, falls prevention, and bone density testing. Corticosteroids have the potential to reduce bone density and increase the risk for fractures.

A nurse admits a patient who has a fracture of the nose that has resulted in a skin tear and involvement of the mucous membranes of the nasal passages. The orthopedic nurse is aware that this description likely indicates which type of fracture? A) Compression B) Compound C) Impacted D) Transverse

Ans: B Feedback: A compound fracture involves damage to the skin or mucous membranes and is also called an open fracture. A compression fracture involves compression of bone and is seen in vertebral fractures. An impacted fracture occurs when a bone fragment is driven into another bone fragment. A transverse fracture occurs straight across the bone shaft.

An elite high school football player has been diagnosed with a shoulder dislocation. The patient has been treated and is eager to resume his role on his team, stating that he is not experiencing pain. What should the nurse emphasize during health education? A) The need to take analgesia regardless of the short-term absence of pain B) The importance of adhering to the prescribed treatment and rehabilitation regimen C) The fact that he has a permanently increased risk of future shoulder dislocations D) The importance of monitoring for intracapsular bleeding once he resumes playing

Ans: B Feedback: Patients who have experienced sports-related injuries are often highly motivated to return to their previous level of activity. Adherence to restriction of activities and gradual resumption of activities needs to be reinforced. Appropriate analgesia use must be encouraged, but analgesia does not necessarily have to be taken in the absence of pain. If healing is complete, the patient does not likely have a greatly increased risk of reinjury. Dislocations rarely cause bleeding after the healing process.

A nurse is caring for a patient who has suffered an unstable thoracolumbar fracture. Which of the following is the priority during nursing care? A) Preventing infection B) Maintaining spinal alignment C) Maximizing function D) Preventing increased intracranial pressure

Ans: B Feedback: Patients with an unstable fracture must have their spine in alignment at all times in order to prevent neurologic damage. This is a greater threat, and higher priority, than promoting function and preventing infection, even though these are both valid considerations. Increased ICP is not a high risk.

Which of the following is the most appropriate nursing intervention to facilitate healing in a patient who has suffered a hip fracture? A) Administer analgesics as required. B) Place a pillow between the patient's legs when turning. C) Maintain prone positioning at all times. D) Encourage internal and external rotation of the affected leg.

Ans: B Feedback: Placing a pillow between the patient's legs when turning prevents adduction and supports the patient's legs. Administering analgesics addresses pain but does not directly protect bone remodeling and promote healing. Rotation of the affected leg can cause dislocation and must be avoided. Prone positioning does not need to be maintained at all times.

The orthopedic nurse should assess for signs and symptoms of Volkmann's contracture if a patient has fractured which of the following bones? A) Femur B) Humerus C) Radial head D) Clavicle

Ans: B Feedback: The most serious complication of a supracondylar fracture of the humerus is Volkmann's ischemic contracture, which results from antecubital swelling or damage to the brachial artery. This complication is specific to humeral fractures.

A patient has sustained a long bone fracture and the nurse is preparing the patient's care plan. Which of the following should the nurse include in the care plan? A) Administer vitamin D and calcium supplements as ordered. B) Monitor temperature and pulses of the affected extremity. C) Perform passive range of motion exercises as tolerated. D) Administer corticosteroids as ordered.

Ans: B Feedback: The nurse should include monitoring for sufficient blood supply by assessing the color, temperature, and pulses of the affected extremity. Weight-bearing exercises are encouraged, but passive ROM exercises have the potential to cause pain and inhibit healing. Corticosteroids, vitamin D, and calcium are not normally administered.

A patient with a simple arm fracture is receiving discharge education from the nurse. What would the nurse instruct the patient to do? A) Elevate the affected extremity to shoulder level when at rest. B) Engage in exercises that strengthen the unaffected muscles. C) Apply topical anesthetics to accessible skin surfaces as needed. D) Avoid using analgesics so that further damage is not masked.

Ans: B Feedback: The nurse will encourage the patient to engage in exercises that strengthen the unaffected muscles. Comfort measures may include appropriate use of analgesics and elevation of the affected extremity to the heart level. Topical anesthetics are not typically used.

A nurse's assessment of a patient's knee reveals edema, tenderness, muscle spasms, and ecchymosis. The patient states that 2 days ago he ran 10 miles and now it "really hurts to stand up." The nurse should plan care based on the belief that the patient has experienced what? A) A first-degree strain B) A second-degree strain C) A first-degree sprain D) A second-degree sprain

Ans: B Feedback: A second-degree strain involves tearing of muscle fibers and is manifested by notable loss of load-bearing strength with accompanying edema, tenderness, muscle spasm, and ecchymosis. A first-degree strain reflects tearing of a few muscle fibers and is accompanied by minor edema, tenderness, and mild muscle spasm, without noticeable loss of function. However, this patient states a loss of function. A sprain normally involves twisting, which is inconsistent with the patient's overuse injury.

A school nurse is assessing a student who was kicked in the shin during a soccer game. The area of the injury has become swollen and discolored. The triage nurse recognizes that the patient has likely sustained what? A) Sprain B) Strain C) Contusion D) Dislocation

Ans: C Feedback: A contusion is a soft-tissue injury that results in bleeding into soft tissues, creating a hematoma and ecchymosis. A sprain is an injury to ligaments caused by wrenching or twisting. A strain is a "muscle pull" from overuse, overstretching, or excessive stress. A dislocation is a condition in which the articular surfaces of the bones forming a joint are no longer in anatomic contact. Because the injury is not at the site of a joint, the patient has not experienced a sprain, strain, or dislocation.

A nurse is caring for a patient who has suffered a hip fracture and who will require an extended hospital stay. The nurse should ensure that the patient does which of the following in order to prevent common complications associated with a hip fracture? A) Avoid requesting analgesia unless pain becomes unbearable. B) Use supplementary oxygen when transferring or mobilizing. C) Increase fluid intake and perform prescribed foot exercises. D) Remain on bed rest for 14 days or until instructed by the orthopedic surgeon.

Ans: C Feedback: Deep vein thrombosis (DVT) is among the most common complications related to a hip fracture. To prevent DVT, the nurse encourages intake of fluids and ankle and foot exercises. The patient should not be told to endure pain; a proactive approach to pain control should be adopted. While respiratory complications commonly include atelectasis and pneumonia, the use of deep-breathing exercises, changes in position at least every 2 hours, and the use of incentive spirometry help prevent the respiratory complications more than using supplementary oxygen. Bed rest may be indicated in the short term, but is not normally required for 14 days.

A nurse is performing a shift assessment on an elderly patient who is recovering after surgery for a hip fracture. The nurse notes that the patient is complaining of chest pain, has an increased heart rate, and increased respiratory rate. The nurse further notes that the patient is febrile and hypoxic, coughing, and producing large amounts of thick, white sputum. The nurse recognizes that this is a medical emergency and calls for assistance, recognizing that this patient is likely demonstrating symptoms of what complication? A) Avascular necrosis of bone B) Compartment syndrome C) Fat embolism syndrome D) Complex regional pain syndrome

Ans: C Feedback: Fat embolism syndrome occurs most frequently in young adults and elderly patients who experience fractures of the proximal femur (i.e., hip fracture). Presenting features of fat embolism syndrome include hypoxia, tachypnea, tachycardia, and pyrexia. The respiratory distress response includes tachypnea, dyspnea, wheezes, precordial chest pain, cough, large amounts of thick, white sputum, and tachycardia. Avascular necrosis (AVN) occurs when the bone loses its blood supply and dies. This does not cause coughing. Complex regional pain syndrome does not have cardiopulmonary involvement.

Six weeks after an above-the-knee amputation (AKA), a patient returns to the outpatient office for a routine postoperative checkup. During the nurse's assessment, the patient reports symptoms of phantom pain. What should the nurse tell the patient to do to reduce the discomfort of the phantom pain? A) Apply intermittent hot compresses to the area of the amputation. B) Avoid activity until the pain subsides. C) Take opioid analgesics as ordered. D) Elevate the level of the amputation site.

Ans: C Feedback: Opioid analgesics may be effective in relieving phantom pain. Heat, immobility, and elevation are not noted to relieve this form of pain.

A young patient is being treated for a femoral fracture suffered in a snowboarding accident. The nurse's most recent assessment reveals that the patient is uncharacteristically confused. What diagnostic test should be performed on this patient? A) Electrolyte assessment B) Electrocardiogram C) Arterial blood gases D) Abdominal ultrasound

Ans: C Feedback: Subtle personality changes, restlessness, irritability, or confusion in a patient who has sustained a fracture are indications for immediate arterial blood gas studies due to the possibility of fat embolism syndrome. This assessment finding does not indicate an immediate need for electrolyte levels, an ECG, or abdominal ultrasound.

A nurse is preparing to discharge an emergency department patient who has been fitted with a sling to support her arm after a clavicle fracture. What should the nurse instruct the patient to do? A) Elevate the arm above the shoulder 3 to 4 times daily. B) Avoid moving the elbow, wrist, and fingers until bone remodeling is complete. C) Engage in active range of motion using the affected arm. D) Use the arm for light activities within the range of motion.

Ans: D Feedback: A patient with a clavicle fracture may use a sling to support the arm and relieve the pain. The patient may be permitted to use the arm for light activities within the range of comfort. The patient should not elevate the arm above the shoulder level until the ends of the bones have united, but the nurse should encourage the patient to exercise the elbow, wrist, and fingers.

Radiographs of a boy's upper arm show that the humerus appears to be fractured on one side and slightly bent on the other. This diagnostic result suggests what type of fracture? A) Impacted B) Compound C) Compression D) Greenstick

Ans: D Feedback: Greenstick fractures are an incomplete fracture that results in the bone being broken on one side, while the other side is bent. This is not characteristic of an impacted, compound, or compression fracture.

An emergency department patient is diagnosed with a hip dislocation. The patient's family is relieved that the patient has not suffered a hip fracture, but the nurse explains that this is still considered to be a medical emergency. What is the rationale for the nurse's statement? A) The longer the joint is displaced, the more difficult it is to get it back in place. B) The patient's pain will increase until the joint is realigned. C) Dislocation can become permanent if the process of bone remodeling begins. D) Avascular necrosis may develop at the site of the dislocation if it is not promptly resolved.

Ans: D Feedback: If a dislocation or subluxation is not reduced immediately, avascular necrosis (AVN) may develop. Bone remodeling does not take place because a fracture has not occurred. Realignment does not become more difficult with time and pain would subside with time, not become worse.

A patient is brought to the emergency department by ambulance after stepping in a hole and falling. While assessing him the nurse notes that his right leg is shorter than his left leg; his right hip is noticeably deformed and he is in acute pain. Imaging does not reveal a fracture. Which of the following is the most plausible explanation for this patient's signs and symptoms? A) Subluxated right hip B) Right hip contusion C) Hip strain D) Traumatic hip dislocation

Ans: D Feedback: Signs and symptoms of a traumatic dislocation include acute pain, change in positioning of the joint, shortening of the extremity, deformity, and decreased mobility. A subluxation would cause moderate deformity, or possibly no deformity. A contusion or strain would not cause obvious deformities.

A nurse is preparing to discharge a patient from the emergency department after receiving treatment for an ankle sprain. While providing discharge education, the nurse should encourage which of the following? A) Apply heat for the first 24 to 48 hours after the injury. B) Maintain the ankle in a dependent position. C) Exercise hourly by performing rotation exercises of the ankle. D) Keep an elastic compression bandage on the ankle.

Ans: D Feedback: Treatment of a sprain consists of resting and elevating the affected part, applying cold, and using a compression bandage. After the acute inflammatory stage (usually 24 to 48 hours after injury), heat may be applied intermittently. Rotation exercises would likely be painful.

A patient had an above-the-knee amputation of the left leg related to complications from peripheral vascular disease (PVD). The nurse enters the patient's room and observes the dressing and bed covers saturated with blood. What is the first action by the nurse? a. Notify the healthcare provider b. Apply a tourniquet c. Use skin clips to close the wound d. Reinforce the dressing

Apply a tourniquet

A patient sustains a fracture of the arm. When does the nurse anticipate pendulum exercise should begin? a. As soon as tolerated, after a reasonable period of immobilization b. In 2 to 3 weeks, when callus ossification prevents easy movements of bony fragments c. In about 4 to 5 weeks, after new bone is well established d. In 2 to 3 months, after normal activities are resumed

As soon as tolerated, after a reasonable period of immobilization

A fragment of the bone is pulled off by a ligament or tendon

Avulsion

close fracture

Broken bone that does not break through the skin

List three early and serious complications associated with bed rest and reduced skeletal muscle contractions for a patient with an open fracture.

Deep vein thrombosis, thromboembolism, and pulmonary embolus.

a condition in which the articular surfaces of the distal and proximal bones that form the joint are no longer in anatomic alignment.

Dislocations

Collateral Ligament Injury treatment

Early management includes PRICE. The joint is evaluated for fracture. Hemarthrosis (bleeding into the joint) may develop, contributing to the pain; should this occur, the joint fluid may be aspirated to relieve pressure.

A patient sustained an open fracture of the femur 24 hours ago. While assessing the patient, the nurse observes the patient is having difficulty breathing, and oxygen saturation decreases to 88% from a previous 99%. What does the nurse determine is likely occurring with this patient? a. Spontaneous pneumothorax b. Cardiac tamponade c. Pneumonia d. Fat emboli

Fat emboli

While riding a bicycle on a narrow road, the patient was hit from behind and thrown into a ditch, sustaining a pelvic fracture. What complications does the nurse monitor for that are common to pelvic fractures? a. Paresthesia and ischemia b. Hemorrhage and shock c. Paralytic ileus and a lacerated urethra d. Thrombophlebitis and infection

Hemorrhage and shock

The nurse is monitoring a patient who sustained an open fracture of the left hip. What type of shock should the nurse recognize can occur with this type of injury? a. Cardiogenic b. Hypovolemic c. Neurogenic d. Septicemic

Hypovolemic

A patient has suffered a femoral shaft fracture in an industrial accident. What is an immediate nursing concern for this patient? a. Hypovolemic shock b. Infection c. Knee and hip dislocation d. Pain resulting from muscle spasm

Hypovolemic shock

A patient sustains an open fracture with extensive soft tissue damage. The nurse determines that this fracture would be classified as what grade? a. I b. II c. III d. IV

III

Collateral Ligament Injury

Injury to these ligaments occurs when the foot is firmly planted and the knee is struck—either medially, causing stretching and tearing injury to the lateral collateral ligament, or laterally, causing stretching and tearing injury to the medial collateral ligament.

_______ epicondylitis (i.e., golfer's elbow) is consistent with repetitive wrist flexion and pronation of the forearm. Extreme tenderness occurs at the medial epicondyle

Medial

A patient is in early shock from a fracture. What five activities are involved in the treatment?

Stabilizing the fracture to prevent further hemorrhage, restoring blood volume and circulation, relieving the patient's pain, providing proper immobilization,and protecting against further injury

a rip in a tendon that connects one of the rotator muscles to the humeral head.

The rotator cuff - stabilizes the humeral head and is composed of four muscles and their tendons that include the supraspinatus, infraspinatus, teres minor, and subscapularis muscles

Cruciate Ligament Injury

These ligaments cross each other in the center of the knee. Injury occurs when the foot is firmly planted and the leg sustains direct force, either forward or backward. *The injured person may report feeling and hearing a "pop" in the knee with this injury.

A patient falls while skiing and sustains a supracondylar fracture of the humerus. What serious complication of this injury should the nurse monitor for? a. Hemarthrosis b. Paresthesia c. Malunion d. Volkmann's ischemic contracture

Volkmann's ischemic contracture

A patient sustains an open fracture of the left arm after an accident at the roller skating rink. What does emergency management of this fracture involve? (Select all that apply.) a. Cover the area with a clean dressing if the fracture is open b. Immobilize the affected site c. Splint the injured limb d. Have the patient demonstrate mobility of the arm e. Wrap the arm in a compression bandage

a, b, c

A nurse is caring for a patient who has had an amputation. What interventions can the nurse provide to foster a positive self-image? (Select all that apply.) a. Encouraging the patient to care for the residual limb b. Allowing the expression of grief c. Encourage the patient to have family and friends view the residual limb to decrease self-consciousness d. Encouraging family and friends to refrain from visiting temporarily because this may increase the patient's embarrassment e. Introducing the patient to local amputee support groups

a. Encouraging the patient to care for the residual limb b. Allowing the expression of grief e. Introducing the patient to local amputee support groups

The nurse is caring for a patient who sustained an open fracture of the right femur in an automobile crash. What does the nurse recognize is the most serious complication of an open fracture? a. Infection b. Muscle atrophy caused by loss of supporting bone structure c. Necrosis of adjacent soft tissue caused by blood loss d. Nerve damage

a. Infection

The residual limb should never be placed on a pillow to avoid ________.

flexion contracture of the hip

Rib fracture treatment

local block and epidural catheter pain control so they can breathe

The femur fracture that commonly leads to avascular necrosis or nonunion due to an abundant supply of blood vessels in the area is a fracture of the ________.

neck

Patients with open fractures risk three major complications: ________, ________, and ________.

osteomyelitis, tetanus, gas gangrene

strain vs sprain

strain: musculo-tendon sprain: ligamentous

With an open fracture, the wound is covered with a sterile dressing to prevent contamination of deeper tissues. No attempt is made to reduce the fracture, even if one of the bone fragments is protruding through the wound. Splints are applied for immobilization

true

medical management osteoporosis

- calcium intake of 100 to 1200 mg - The recommended vitamin D intake 600 (young) 800 IU (older) - Regular weight-bearing exercise promotes bone formation

On a visit to the family physician, a client is diagnosed with a bunion on the lateral side of the great toe, at the metatarsophalangeal joint. Which statement should the nurse include in the teaching session? a) "Bunions are congenital and can't be prevented." b) "Bunions may result from wearing shoes that are too big, causing friction when the shoes slip back and forth." c) "Bunions are caused by a metabolic condition called gout." d) "Some bunions are congenital; others are caused by wearing shoes that are too short or narrow."

"Some bunions are congenital; others are caused by wearing shoes that are too short or narrow." Explanation: Bunions may be congenital or may be acquired by wearing shoes that are too short or narrow, which increases pressure on the bursa at the metatarsophalangeal joint. Acquired bunions can be prevented. Wearing shoes that are too big may cause other types of foot trauma but not bunions. Gout doesn't cause bunions. Although a client with gout may have pain in the big toe, such pain doesn't result from a bunion.

A nurse is educating a patient diagnosed with osteomalacia. Which of the following statements by the nurse is appropriate? a) "You will need to avoid foods high in phosphorus, and vitamin D." b) "You will need to engage in vigorous exercise three times a week for 30 minutes." c) "You will need to decrease the amount of dairy products consumed." d) "You may need to be evaluated for an underlying cause, such as renal failure."

"You may need to be evaluated for an underlying cause, such as renal failure." Explanation: The patient may need to be evaluated for an underlying cause. If an underlying cause is discovered, that will guide the medical treatment. The patient needs to maintain an adequate to increased supply of calcium, phosphorus, and vitamin D. Dairy products are a good source of calcium. The patient is at risk for pathological fractures and therefore should not engage in vigorous exercise.

A nurse is educating a patient diagnosed with osteomalacia. Which of the following statements by the nurse is appropriate? a) "You will need to engage in vigorous exercise three times a week for 30 minutes." b) "You will need to decrease the amount of dairy products consumed." c) "You may need to be evaluated for an underlying cause, such as renal failure." d) "You will need to avoid foods high in phosphorus, and vitamin D."

"You may need to be evaluated for an underlying cause, such as renal failure." Explanation: The patient may need to be evaluated for an underlying cause. If an underlying cause is discovered, that will guide the medical treatment. The patient needs to maintain an adequate to increased supply of calcium, phosphorus, and vitamin D. Dairy products are a good source of calcium. The patient is at risk for pathological fractures and therefore should not engage in vigorous exercise.

Arthroscopy

- A large-bore needle is inserted, and the joint is distended with saline. - allows direct visualization of a joint through the use of a fiberoptic endoscope. Thus, it is a useful adjunct to diagnosing joint disorders.

arthrography is used to identify the cause of any unexplained joint pain and progression of joint disease.

- A radiopaque contrast agent or air is injected into the joint cavity to visualize the joint structures, such as the ligaments, cartilage, tendons, and joint capsule. - The joint is put through its range of motion to distribute the contrast agent while a series of x-rays are obtained. - If a tear is present, the contrast agent leaks out of the joint and is evident on the x-ray image

Bisphosphonates Key Nursing Considerations

- Adequate calcium and vitamin D intake is needed to assure maximum effect; however, these supplements should not be taken at the same time as the bisphosphonates -

Bisphosphonates to treat osteoporosis

- Alendronate (Fosamax) - Risedronate (Actonel) - Ibandronate (Boniva) - Zoledronic acid (Reclast)

diarthrosis joints:

- Ball-and-socket joints - Hinge joints permit - Saddle joints allow -Pivot joints - Gliding joints

treatment for DVT

- Blood thinners -Clot busters -Compression stockings

traction principles

- Ensure continuous traction - Maintain counter traction - Body alignment - Apply exact amount of weight prescribed - Ropes move freely through pulleys - Weight hangs freely

Nursing Interventions Arthrocentesis

- Hair may need to be removed from the site - Ice may be prescribed for the first 24 to 48 hours postprocedure - symptoms of complications, infection and bleeding

The nurse must monitor for signs and symptoms of dislocation of the prosthesis, which include:

- Increased pain at the surgical site, swelling, and immobilization - Acute groin pain in the affected hip or increased discomfort - Shortening of the affected extremity - Abnormal external or internal rotation of the affected extremity - Restricted ability or inability to move the leg - Reported "popping" sensation in the hip

spica cast care

- Keep the cast dry. - The edges of the cast should be padded as needed to prevent skin irritation - monitor bowel sounds every 4 to 8 hours - position patient frequently (on their side)

Nursing Management of the Patient With a Body or Spica Cast

- Nursing responsibilities include preparing and positioning the patient, assisting with skin care and hygiene, and monitoring for cast syndrome

Zoledronic acid (Reclast)

- Prevention and treatment of osteoporosis in women who are postmenopausal - Administer IV once yearly for osteoporosis treatment or once every 2 years for osteoporosis prevention

Raloxifene (Evista)

- Prevention and treatment of osteoporosis in women who are postmenopausal, particularly those with breast cancer - Administer PO once daily. - May be given in tandem with calcium and vitamin D - Side effects include hot flashes and leg cramps - Adverse effects include VTE formation

Ballottement sign

- The medial and lateral aspects of the extended knee are milked firmly in a downward motion. - The examiner pushes the patella toward the femur and observes for fluid return to the region superior to the patella. - When larger amounts of fluid are present, the patella elevates, there is visible return of fluid to the region directly superior to the patella, and the ballottement test is positive

Nursing Management of the Patient With an Immobilized Upper Extremity

- The unaffected arm will assume all upper extremity activities. - To control swelling, the immobilized arm is elevated above heart level with a pillow. - When the patient is lying down, the arm is elevated so that each joint is positioned higher than the preceding proximal joint (e.g., elbow higher than the shoulder, hand higher than the elbow).

Denosumab (Prolia)

- Treatment of osteoporosis in men and women who are postmenopausal at high risk of fracture - women with osteoporosis and breast cancer receiving aromatase inhibitors and for men with osteoporosis and prostate cancer receiving gonadotropin-reducing hormones

Ibandronate (Boniva)

- Treatment of osteoporosis in women who are postmenopausal - IV dosing may be good option for patients either intolerant of PO bisphosphonates or nonadherent to prescribed therapy

Weight-bearing activity vs weight-resistance exercise

- Weight-bearing activity, which supports bone maintenance, is any activity done while a person is on their feet that works a person's bones and muscles against gravity (e.g., walking, tennis). - Weight-resistance exercise uses weights or resistance to strengthen muscles.

Hallux Valgus

- a deformity in which the great toe deviates laterally - Acute bursitis symptoms include a reddened area, edema, and tenderness.

Hammer Toe

- a flexion deformity of the interphalangeal joint, which may involve several toes - Tight socks or shoes may push an overlying toe back into the line of the other toes.

Morton Neuroma

- a swelling of the third (lateral) branch of the median plantar nerve - The result is a throbbing, burning pain in the foot that is usually relieved with rest and massage.

Contracture of muscle

- a tightening or shortening of muscles. - It causes joint stiffness and can happen in any joint. - patient may get contractures from having to stay in bed for a long time

ganglion hand

- appears as a round, firm, cystic swelling, usually on the dorsum of the wrist - The swelling is locally tender and may cause an aching pain.

bone scan

- detect metastatic and primary bone tumors, osteomyelitis, some fractures, and aseptic necrosis, and to monitor the progression of degenerative bone diseases. * requires the injection of a radioisotope through an IV line

Calcitriol functions

- increase the amount of calcium in the blood by promoting absorption of calcium from the gastrointestinal tract. - mineralization of osteoid tissue.

Medications used to treat early impingement syndrome include:

- oral NSAIDs or intra-articular injections of corticosteroids. - Application of superficial cold or heat may subjectively improve patients' symptoms

The "6 Ps" indicative of symptoms of neurovascular compromise are

- pain - poikilothermia (i.e., takes on the ambient temperature) - pallor - pulselessness - paresthesia - paralysis

parathyroid hormone function bones

- parathyroid hormone stimulates the release of calcium from large calcium stores in the bones into the bloodstream. This increases bone destruction and decreases the formation of new bone. - Kidneys, parathyroid hormone reduces loss of calcium in urine.

PTH function in bone

- regulates the concentration of calcium in the blood, in part by promoting movement of calcium from the bone. - In response to low calcium levels in the blood, increased levels of PTH prompt the mobilization of calcium, the demineralization of bone, and the formation of bone cysts.

psychological components of cast syndrome

- similar to a claustrophobic reaction - acute anxiety reaction characterized by behavioral changes and autonomic responses (e.g., increased respiratory rate, diaphoresis, dilated pupils, increased heart rate, elevated blood pressure).

external fixation devices

- used to manage fractures with soft tissue damage. - Complicated fractures of the humerus, forearm, femur, tibia, and pelvis are managed with external skeletal fixators.

skeletal traction

- used when continuous traction is desired to immobilize, position, and align a fracture of the femur, tibia, and cervical spine. - when traction is to be maintained for a significant amount of time

Clinical Manifestations septic arthritis

- warm, painful, swollen joint with decreased range of motion. - Systemic chills, fever, and leukocytosis are sometimes presen

Circulatory assessment consists of:

-peripheral pulses, color, capillary refill, and temperature of the fingers or toes. - Manifestations of deep vein thrombosis (DVT), which include unilateral calf tenderness, warmth, redness, and swelling.

Fracture healing process

1. hematoma formation 2. fibrocartilage callus formation 3. bony callus formation 4. bone remodeling

A nutritious diet is particularly important in older adults. Adequate intake of calcium and vitamin D is promoted. Because sunlight is necessary for synthesizing vitamin D, patients should be encouraged to spend some time in the sun, for at least ___ minutes and up to an hour daily

10

After total hip replacement patients should be instructed to dorsi- and plantar flex the ankles and the toes ____ to ____ times every half hour while awake

10 to 20

Patients who experience a fracture of the humeral neck are advised that healing will take an average of ________ weeks, with restricted vigorous activity for an additional ________ weeks.

10, 4

Daily intake of approximately _____________ to __________ mg of calcium is essential to maintaining adult bone mass.

1000 to 1200

The nurse is caring for a patient with a total hip replacement. How should the nurse assist the patient to turn? a. 45 degrees onto the unoperated side if the affected hip is kept abducted b. Assist from the prone to the supine position only, and the patient must keep the affected hip extended and abducted c. Any comfortable position is acceptable as long as the affected leg is extended d. Assist to the operative side if the affected hip remains extended

45 degrees onto the unoperated side if the affected hip is kept abducted

The nurse is caring for patient with a hip fracture. The physician orders the patient to start on a bisphosphonate. Which medication would the nurse document as given? a) Raloxifene (Evista) b) Alendronate (Fosamax) c) Teriparatide (Forteo) d) Denosumab (Prolia)

Alendronate (Fosamax) Explanation: Alendronate (Fosamax) is a bisphosphonate medication. Raloxifene (Evista) is a selective estrogen receptor modulator. Terparatide (Forteo) is an anabolic agent, and denosumab (Prolia) is a monoclonal antibody agent.

A patient was fitted with an arm cast after fracturing her humerus. Twelve hours after the application of the cast, the patient tells the nurse that her arm hurts. Analgesics do not relieve the pain. What would be the most appropriate nursing action? A) Prepare the patient for opening or bivalving of the cast. B) Obtain an order for a different analgesic. C) Encourage the patient to wiggle and move the fingers. D) Petal the edges of the patient's cast.

Ans: A Feedback: Acute compartment syndrome involves a sudden and severe decrease in blood flow to the tissues distal to an area of injury that results in ischemic necrosis if prompt, decisive intervention does not occur. Removing or bivalving the cast is necessary to relieve pressure. Ordering different analgesics does not address the underlying problem. Encouraging the patient to move the fingers or perform range-of-motion exercises will not treat or prevent compartment syndrome. Petaling the edges of a cast with tape prevents abrasions and skin breakdown, not compartment syndrome.

A patient tells the nurse that he has pain and numbness to his thumb, first finger, and second finger of the right hand. The nurse discovers that the patient is employed as an auto mechanic, and that the pain is increased while working. This may indicate that the patient could possibly have what health problem? A) Carpel tunnel syndrome B) Tendonitis C) Impingement syndrome D) Dupuytren's contracture

Ans: A Feedback: Carpel tunnel syndrome may be manifested by numbness, pain, paresthesia, and weakness along the median nerve. Tendonitis is inflammation of muscle tendons. Impingement syndrome is a general term that describes all lesions that involve the rotator cuff of the shoulder. Dupuytren's contracture is a slowly progressive contracture of the palmar fascia.

A 25-year-old man is involved in a motorcycle accident and injures his arm. The physician diagnoses the man with an intra-articular fracture and splints the injury. The nurse implements the teaching plan developed for this patient. What sequela of intra-articular fractures should the nurse describe regarding this patient? A) Post-traumatic arthritis B) Fat embolism syndrome (FES) C) Osteomyelitis D) Compartment syndrome

Ans: A Feedback: Intra-articular fractures often lead to post-traumatic arthritis. Research does not indicate a correlation between intra-articular fractures and FES, osteomyelitis, or compartment syndrome.

A nurse is assessing a patient who reports a throbbing, burning sensation in the right foot. The patient states that the pain is worst during the day but notes that the pain is relieved with rest. The nurse should recognize the signs and symptoms of what health problem? A) Morton's neuroma B) Pescavus C) Hallux valgus D) Onychocryptosis

Ans: A Feedback: Morton's neuroma is a swelling of the third (lateral) branch of the median plantar nerve, which causes a throbbing, burning pain, usually relieved with rest. Pescavus refers to a foot with an abnormally high arch and a fixed equinus deformity of the forefoot. Hallux valgus (bunion) is a deformity in which the great toe deviates laterally and there is a marked prominence of the medial aspect of the first metatarsal-phalangeal joint and exostosis. Onychocryptosis (ingrown toenail) occurs when the free edge of a nail plate penetrates the surrounding skin, laterally or anteriorly.

A 32-year-old patient comes to the clinic complaining of shoulder tenderness, pain, and limited movement. Upon assessment the nurse finds edema. An MRI shows hemorrhage of the rotator cuff tendons and the patient is diagnosed with impingement syndrome. What action should the nurse recommend in order to promote healing? A) Support the affected arm on pillows at night. B) Take prescribed corticosteroids as ordered. C) Put the shoulder through its full range of motion 3 times daily. D) Keep the affected arm in a sling for 2 to 4 weeks.

Ans: A Feedback: The patient should support the affected arm on pillows while sleeping to keep from turning onto the shoulder. Corticosteroids are not commonly prescribed and a sling is not normally necessary. ROM exercises are indicated, but putting the arm through its full ROM may cause damage during the healing process.

A nurse is collaborating with the physical therapist to plan the care of a patient with osteomyelitis. What principle should guide the management of activity and mobility in this patient? A) Stress on the weakened bone must be avoided. B) Increased heart rate enhances perfusion and bone healing. C) Bed rest results in improved outcomes in patients with osteomyelitis. D) Maintenance of baseline ADLs is the primary goal during osteomyelitis treatment

Ans: A Feedback: The patient with osteomyelitis has bone that is weakened by the infective process and must be protected by avoidance of stress on the bone.This risk guides the choice of activity in a patient with osteomyelitis. Bed rest is not normally indicated, however. Maintenance of prediagnosis ADLs may be an unrealistic short-term goal for many patients.

An 80-year-old man in a long-term care facility has a chronic leg ulcer and states that the area has become increasingly painful in recent days. The nurse notes that the site is now swollen and warm to the touch. The patient should undergo diagnostic testing for what health problem? A) Osteomyelitis B) Osteoporosis C) Osteomalacia D) Septic arthritis

Ans: A Feedback: When osteomyelitis develops from the spread of an adjacent infection, no signs of septicemia are present, but the area becomes swollen, warm, painful, and tender to touch. Osteoporosis is the most prevalent bone disease in the world. Osteomalacia is a metabolic bone disease characterized by inadequate mineralization of bone. Septicarthritis occurs when joints become infected through spread of infection from other parts of the body (hematogenous spread) or directly through trauma or surgical instrumentation.

A nurse is caring for an adult patient diagnosed with a back strain. What health education should the nurse provide to this patient? A) Avoid lifting more than one-third of body weight without assistance. B) Focus on using back muscles efficiently when lifting heavy objects. C) Lift objects while holding the object a safe distance from the body. D) Tighten the abdominal muscles and lock the knees when lifting of an object.

Ans: A Feedback: The nurse will instruct the patient on the safe and correct way to lift objects—using the strong quadriceps muscles of the thighs, with minimal use of the weak back muscles. To prevent recurrence of acute low back pain, the nurse may instruct the patient to avoid lifting more than one-third of his weight without help. The patient should be informed to place the feet a hip-width apart to provide a wide base of support, the person should bend the knees, tighten the abdominal muscles, and lift the object close to the body with a smooth motion, avoiding twisting and jerking.

A patient was brought to the emergency department after a fall. The patient is taken to the operating room to receive a right hip prosthesis. In the immediate postoperative period, what health education should the nurse emphasize? A) "Make sure you don't bring your knees close together." B) "Try to lie as still as possible for the first few days." C) "Try to avoid bending your knees until next week." D) "Keep your legs higher than your chest whenever you can."

Ans: A Feedback: After receiving a hip prosthesis, the affected leg should be kept abducted. Mobility should be encouraged within safe limits. There is no need to avoid knee flexion and the patient's legs do not need to be higher than the level of the chest.

The nurse is helping to set up Buck's traction on an orthopedic patient. How often should the nurse assess circulation to the affected leg? A) Within 30 minutes, then every 1 to 2 hours B) Within 30 minutes, then every 4 hours C) Within 30 minutes, then every 8 hours D) Within 30 minutes, then every shift

Ans: A Feedback: After skin traction is applied, the nurse assesses circulation of the foot or hand within 15 to 30 minutes and then every 1 to 2 hours.

The nursing care plan for a patient in traction specifies regular assessments for venous thromboembolism (VTE). When assessing a patient's lower limbs, what sign or symptom is suggestive of deep vein thrombosis (DVT)? A) Increased warmth of the calf B) Decreased circumference of the calf C) Loss of sensation to the calf D) Pale-appearing calf

Ans: A Feedback: Signs of DVT include increased warmth, redness, swelling, and calf tenderness. These findings are promptly reported to the physician for definitive evaluation and therapy. Signs and symptoms of a DVT do not include a decreased circumference of the calf, a loss of sensation in the calf, or a pale-appearing calf.

A nurse is caring for a patient who is postoperative day 1 right hip replacement. How should the nurse position the patient? A) Keep the patient's hips in abduction at all times. B) Keep hips flexed at no less than 90 degrees. C) Elevate the head of the bed to high Fowler's. D) Seat the patient in a low chair as soon as possible.

Ans: A Feedback: The hips should be kept in abduction by an abductor pillow. Hips should not be flexed more than 90 degrees, and the head of bed should not be elevated more than 60 degrees. The patient's hips should be higher than the knees; as such, high seat chairs should be used.

A patient broke his arm in a sports accident and required the application of a cast. Shortly following application, the patient complained of an inability to straighten his fingers and was subsequently diagnosed with Volkmann contracture. What pathophysiologic process caused this complication? A) Obstructed arterial blood flow to the forearm and hand B) Simultaneous pressure on the ulnar and radial nerves C) Irritation of Merkel cells in the patient's skin surfaces D) Uncontrolled muscle spasms in the patient's forearm

Ans: A Feedback: Volkmann contracture occurs when arterial blood flow is restricted to the forearm and hand and results in contractures of the fingers and wrist. It does not result from nerve pressure, skin irritation, or spasms.

A patient has been admitted to the medical unit for the treatment of Paget's disease. When reviewing the medication administration record, the nurse should anticipate what medications? Select all that apply. A) Calcitonin B) Bisphosphonates C) Alkaline phosphatase D) Calcium gluconate E) Estrogen

Ans: A, B Feedback: Bisphosphonates are the cornerstone of Paget therapy in that they stabilize the rapid bone turnover. Calcitonin is also used because it retards bone resorption by decreasing the number and availability of osteoclasts. Alkaline phosphatase is a naturally occurring enzyme, not a drug. Calcium gluconate and estrogen are not used in the treatment of Paget's disease.

A nurse is providing care for a patient who has a recent diagnosis of Paget's disease. When planning this patient's nursing care, interventions should address what nursing diagnoses? Select all that apply. A) Impaired Physical Mobility B) Acute Pain C) Disturbed Auditory Sensory Perception D) Risk for Injury E) Risk for Unstable Blood Glucose

Ans: A, B, C, D Feedback: Patient's with Paget's disease are at risk of decreased mobility, pain, hearing loss, and injuries resulting from decreased bone density. Paget's disease does not affect blood glucose levels.

A patient has been admitted to the hospital with a spontaneous vertebral fracture related to osteoporosis. Which of the following nursing diagnoses must be addressed in the plan of care? A) Risk for Aspiration Related to Vertebral Fracture B) Constipation Related to Vertebral Fracture C) Impaired Swallowing Related to Vertebral Fracture D) Decreased Cardiac Output Related to Vertebral Fracture

Ans: B Feedback: Constipation is a problem related to immobility and medications used to treat vertebral fractures. The patient's risks of aspiration, dysphagia, and decreased cardiac output are not necessarily heightened.

A patient has returned to the unit after undergoing limb-sparing surgery to remove a metastatic bone tumor. The nurse providing postoperative care in the days following surgery assesses for what complication from surgery? A) Deficient fluid volume B) Delayed wound healing C) Hypocalcemia D) Pathologic fractures

Ans: B Feedback: Delayed wound healing is a complication of surgery due to tissue trauma from the surgery. Nutritional deficiency is usually due to the effects of chemotherapy and radiation therapy, which may cause weight loss. Pathologic fractures are not a complication of surgery.

A patient with diabetes is attending a class on the prevention of associated diseases. What action should the patient perform to reduce the risk of osteomyelitis? A) Increase calcium and vitamin intake. B) Perform meticulous foot care. C) Exercise 3 to 4 times weekly for at least 30 minutes. D) Take corticosteroids as ordered.

Ans: B Feedback: Diabetic foot ulcers have a high potential for progressing to osteomyelitis. Meticulous foot care can help mitigate this risk. Corticosteroids can exacerbate the risk of osteomyelitis. Increased intake of calcium and vitamins as well as regular exercise are beneficial health promotion exercises, but they do not directly reduce the risk of osteomyelitis.

An older adult patient has fallen in her home and is brought to the emergency department by ambulance with a suspected fractured hip. X-rays confirm a fracture of the left femoral neck. When planning assessments during the patient's presurgical care, the nurse should be aware of the patient's heightened risk of what complication? A) Osteomyelitis B) Avascular necrosis C) Phantom pain D) Septicemia

Ans: B Feedback: Fractures of the neck of the femur may damage the vascular system that supplies blood to the head and the neck of the femur, and the bone may become ischemic. For this reason, AVN is common in patients with femoral neck fractures. Infections are not immediate complications and phantom pain applies to patients with amputations, not hip fractures.

A patient with diabetes has been diagnosed with osteomyelitis. The nurse notes that the patient's right foot is pale and mottled, cool to touch, with a capillary refill of greater than 3 seconds. The nurse should suspect what type of osteomyelitis? A) Hematogenous osteomyelitis B) Osteomyelitis with vascular insufficiency C) Contiguous-focus osteomyelitis D) Osteomyelitis with muscular deterioration

Ans: B Feedback: Osteomyelitis is classified as hematogenous osteomyelitis (i.e., due to blood-borne spread of infection); contiguous-focus osteomyelitis, from contamination from bone surgery, open fracture, or traumatic injury (e.g., gunshot wound); and osteomyelitis with vascular insufficiency, seen most commonly among patients with diabetes and peripheral vascular disease, most commonly affecting the feet. Osteomyelitis with muscular deterioration does not exist.

A patient presents at a clinic complaining of pain in his heel so bad that it inhibits his ability to walk. The patient is subsequently diagnosed with plantar fasciitis. This patient's plan of care should include what intervention? A) Wrapping the affected area in lamb's wool or gauze to relieve pressure B) Gently stretching the foot and the Achilles tendon C) Wearing open-toed shoes at all times D) Applying topical analgesic ointment to plantar surface each morning

Ans: B Feedback: Plantar fasciitis leads to pain that is localized to the anterior medial aspect of the heel and diminishes with gentle stretching of the foot and Achilles tendon. Dressings of any kind are not of therapeutic benefit and analgesic ointments do not address the pathology of the problem. Open-toed shoes are of no particular benefit.

The orthopedic surgeon has prescribed balanced skeletal traction for a patient. What advantage is conferred by balanced traction? A) Balanced traction can be applied at night and removed during the day. B) Balanced traction allows for greater patient movement and independence than other forms of traction. C) Balanced traction is portable and may accompany the patient's movements. D) Balanced traction facilitates bone remodeling in as little as 4 days.

Ans: B Feedback: Often, skeletal traction is balanced traction, which supports the affected extremity, allows for some patient movement, and facilitates patient independence and nursing care while maintaining effective traction. It is not portable, however, and it cannot be removed. Bone remodeling takes longer than 4 days.

A patient is being prepared for a total hip arthroplasty, and the nurse is providing relevant education. The patient is concerned about being on bed rest for several days after the surgery. The nurse should explain what expectation for activity following hip replacement? A) "Actually, patients are only on bed rest for 2 to 3 days before they begin walking with assistance." B) "The physical therapist will likely help you get up using a walker the day after your surgery." C) "Our goal will actually be to have you walking normally within 5 days of your surgery." D) "For the first two weeks after the surgery, you can use a wheelchair to meet your mobility needs."

Ans: B Feedback: Patients post-THA begin ambulation with the assistance of a walker or crutches within a day after surgery. Wheelchairs are not normally utilized. Baseline levels of mobility are not normally achieved until several weeks after surgery, however.

A nurse is assessing a patient who is receiving traction. The nurse's assessment confirms that the patient is able to perform plantar flexion. What conclusion can the nurse draw from this finding? A) The leg that was assessed is free from DVT. B) The patient's tibial nerve is functional. C) Circulation to the distal extremity is adequate. D) The patient does not have peripheral neurovascular dysfunction.

Ans: B Feedback: Plantar flexion demonstrates function of the tibial nerve. It does not demonstrate the absence of DVT and does not allow the nurse to ascertain adequate circulation. The nurse must perform more assessments on more sites in order to determine an absence of peripheral neurovascular dysfunction.

A nurse is caring for an older adult patient who is preparing for discharge following recovery from a total hip replacement. Which of the following outcomes must be met prior to discharge? A) Patient is able to perform ADLs independently. B) Patient is able to perform transfers safely. C) Patient is able to weight-bear equally on both legs. D) Patient is able to demonstrate full ROM of the affected hip.

Ans: B Feedback: The patient must be able to perform transfers and to use mobility aids safely. Each of the other listed goals is unrealistic for the patient who has undergone recent hip replacement.

A patient has just begun been receiving skeletal traction and the nurse is aware that muscles in the patient's affected limb are spastic. How does this change in muscle tone affect the patient's traction prescription? A) Traction must temporarily be aligned in a slightly different direction. B) Extra weight is needed initially to keep the limb in proper alignment. C) A lighter weight should be initially used. D) Weight will temporarily alternate between heavier and lighter weights.

Ans: B Feedback: The traction weights applied initially must overcome the shortening spasms of the affected muscles. As the muscles relax, the traction weight is reduced to prevent fracture dislocation and to promote healing. Weights never alternate between heavy and light.

A nurse is caring for a patient receiving skeletal traction. Due to the patient's severe limits on mobility, the nurse has identified a risk for atelectasis or pneumonia. What intervention should the nurse provide in order to prevent these complications? A) Perform chest physiotherapy once per shift and as needed. B) Teach the patient to perform deep breathing and coughing exercises. C) Administer prophylactic antibiotics as ordered. D) Administer nebulized bronchodilators and corticosteroids as ordered.

Ans: B Feedback: To prevent these complications, the nurse should educate the patient about performing deep-breathing and coughing exercises to aid in fully expanding the lungs and clearing pulmonary secretions. Antibiotics, bronchodilators, and steroids are not used on a preventative basis and chest physiotherapy is unnecessary and implausible for a patient in traction.

A patient with a total hip replacement is progressing well and expects to be discharged tomorrow. On returning to bed after ambulating, he complains of a new onset of pain at the surgical site. What is the nurse's best action? A) Administer pain medication as ordered. B) Assess the surgical site and the affected extremity. C) Reassure the patient that pain is a direct result of increased activity. D) Assess the patient for signs and symptoms of systemic infection.

Ans: B Feedback: Worsening pain after a total hip replacement may indicate dislocation of the prosthesis. Assessment of pain should include evaluation of the wound and the affected extremity. Assuming he's anxious about discharge and administering pain medication do not address the cause of the pain. Sudden severe pain is not considered normal after hip replacement. Sudden pain is rarely indicative of a systemic infection.

A nurse is planning the care of a patient with osteomyelitis that resulted from a diabetic foot ulcer. The patient requires a transmetatarsal amputation. When planning the patient's postoperative care, which of the following nursing diagnoses should the nurse most likely include in the plan of care? A) Ineffective Thermoregulation B) Risk-Prone Health Behavior C) Disturbed Body Image D) Deficient Diversion Activity

Ans: C Feedback: Amputations present a serious threat to any patient's body image. None of the other listed diagnoses is specifically associated with amputation.

A patient has presented to the emergency department with an injury to the wrist. The patient is diagnosed with a third-degree strain. Why would the physician order an x-ray of the wrist? A) Nerve damage is associated with third-degree strains. B) Compartment syndrome is associated with third-degree strains. C) Avulsion fractures are associated with third-degree strains. D) Greenstick fractures are associated with third-degree strains.

Ans: C Feedback: An x-ray should be obtained to rule out bone injury, because an avulsion fracture (in which a bone fragment is pulled away from the bone by a tendon) may be associated with a third-degree strain. Nerve damage, compartment syndrome, and greenstick fractures are not associated with third-degree strains.

A 20 year-old is brought in by ambulance to the emergency department after being involved in a motorcycle accident. The patient has an open fracture of his tibia. The wound is highly contaminated and there is extensive soft-tissue damage. How would this patient's fracture likely be graded? A) Grade I B) Grade II C) Grade III D) Grade IV

Ans: C Feedback: Open fractures are graded according to the following criteria. Grade I is a clean wound less than 1 cm long. Grade II is a larger wound without extensive soft-tissue damage. Grade III is highly contaminated, has extensive soft-tissue damage, and is the most severe. There is no grade IV fracture.

A nurse is providing care for a patient who has osteomalacia. What major goal will guide the choice of medical and nursing interventions? A) Maintenance of skin integrity B) Prevention of bone metastasis C) Maintenance of adequate levels of activated vitamin D D) Maintenance of adequate parathyroid hormone function

Ans: C Feedback: The primary defect in osteomalacia is a deficiency of activated vitamin D, which promotes calcium absorption from the gastrointestinal tract and facilitates mineralization of bone. Interventions are aimed at resolving the processes underlying this deficiency. Maintenance of skin integrity is important, but is not the primary goal in care. Osteomalacia is not a malignant process. Overproduction (not underproduction) of PTH can cause the disease.

The nurse is caring for a patient who underwent a total hip replacement yesterday. What should the nurse do to prevent dislocation of the new prosthesis? A) Keep the affected leg in a position of adduction. B) Have the patient reposition himself independently. C) Protect the affected leg from internal rotation. D) Keep the hip flexed by placing pillows under the patient's knee.

Ans: C Feedback: Abduction of the hip helps to prevent dislocation of a new hip joint. Rotation and adduction should be avoided. While the hip may be flexed slightly, it shouldn't exceed 90 degrees and maintenance of flexion isn't necessary. The patient may not be capable of safe independent repositioning at this early stage of recovery.

An elderly patient's hip joint is immobilized prior to surgery to correct a femoral head fracture. What is the nurse's priority assessment? A) The presence of leg shortening B) The patient's complaints of pain C) Signs of neurovascular compromise D) The presence of internal or external rotation

Ans: C Feedback: Because impaired circulation can cause permanent damage, neurovascular assessment of the affected leg is always a priority assessment. Leg shortening and internal or external rotation are common findings with a fractured hip. Pain, especially on movement, is also common after a hip fracture.

A 91-year-old patient is slated for orthopedic surgery and the nurse is integrated gerontologic considerations into the patient's plan of care. What intervention is most justified in the care of this patient? A) Administration of prophylactic antibiotics B) Total parenteral nutrition (TPN) C) Use of a pressure-relieving mattress D) Use of a Foley catheter until discharge

Ans: C Feedback: Older adults have a heightened risk of skin breakdown; use of a pressure-reducing mattress addresses this risk. Older adults do not necessarily need TPN and the Foley catheter should be discontinued as soon as possible to prevent urinary tract infections. Prophylactic antibiotics are not a standard infection prevention measure.

A nurse is caring for a patient who has a leg cast. The nurse observes that the patient uses a pencil to scratch the skin under the edge of the cast. How should the nurse respond to this observation? A) Allow the patient to continue to scratch inside the cast with a pencil but encourage him to be cautious. B) Give the patient a sterile tongue depressor to use for scratching instead of the pencil. C) Encourage the patient to avoid scratching, and obtain an order for an antihistamine if severe itching persists. D) Obtain an order for a sedative, such as lorazepam (Ativan), to prevent the patient from scratching.

Ans: C Feedback: Scratching should be discouraged because of the risk for skin breakdown or damage to the cast. Most patients can be discouraged from scratching if given a mild antihistamine, such as diphenhydramine, to relieve itching. Benzodiazepines would not be given for this purpose.

The nurse has identified the diagnosis of Risk for Impaired Tissue Perfusion Related to Deep Vein Thrombosis in the care of a patient receiving skeletal traction. What nursing intervention best addresses this risk? A) Encourage independence with ADLs whenever possible. B) Monitor the patient's nutritional status closely. C) Teach the patient to perform ankle and foot exercises within the limitations of traction. D) Administer clopidogrel (Plavix) as ordered.

Ans: C Feedback: The nurse educates the patient how to perform ankle and foot exercises within the limits of the traction therapy every 1 to 2 hours when awake to prevent DVT. Nutrition is important, but does not directly prevent DVT. Similarly, independence with ADLs should be promoted, but this does not confer significant prevention of DVT, which often affects the lower limbs. Plavix is not normally used for DVT prophylaxis.

A nurse is caring for a patient in skeletal traction. In order to prevent bony fragments from moving against one another, the nurse should caution the patient against which of the following actions? A) Shifting one's weight in bed B) Bearing down while having a bowel movement C) Turning from side to side D) Coughing without splinting

Ans: C Feedback: To prevent bony fragments from moving against one another, the patient should not turn from side to side; however, the patient may shift position slightly with assistance. Bearing down and coughing do not pose a threat to bone union.

A patient presents at the clinic with complaints of morning numbness, cramping, and stiffness in his fourth and fifth fingers. What disease process should the nurse suspect? A) Tendonitis B) A ganglion C) Carpal tunnel syndrome D) Dupuytren's disease

Ans: D Feedback: In cases of Dupuytren's disease, the patient may experience dull, aching discomfort, morning numbness, cramping, and stiffness in the affected fingers. This condition starts in one hand, but eventually both hands are affected. This clinical scenario does not describe tendonitis, a ganglion, or carpal tunnel syndrome.

An orthopedic nurse is caring for a patient who is postoperative day one following foot surgery. What nursing intervention should be included in the patient's subsequent care? A) Dressing changes should not be performed unless there are clear signs of infection. B) The surgical site can be soaked in warm bath water for up to 5 minutes. C) The surgical site should be cleansed with hydrogen peroxide once daily. D) The foot should be elevated in order to prevent edema.

Ans: D Feedback: Pain experienced by patients who undergo foot surgery is related to inflammation and edema. To control the anticipated edema, the foot should be elevated on several pillows when the patient is sitting or lying. Regular dressing changes are performed and the wound should be kept dry. Hydrogen peroxide is not used to cleanse surgical wounds.

A nurse is providing a class on osteoporosis at the local seniors' center. Which of the following statements related to osteoporosis is most accurate? A) Osteoporosis is categorized as a disease of the elderly. B) A nonmodifiable risk factor for osteoporosis is a person's level of activity. C) Secondary osteoporosis occurs in women after menopause. D) Slow discontinuation of corticosteroid therapy can halt the progression of the osteoporosis.

Ans: D Feedback: When corticosteroid therapy is discontinued, the progression of osteoporosis is halted, but restoration of lost bone mass does not occur. Osteoporosis is not a disease of the elderly because its onset occurs earlier in life, when bone mass peaks and then begins to decline. A person's level of physical activity is a modifiable factor that influences peak bone mass. Lack of activity increases the risk for the development of osteoporosis. Primary osteoporosis occurs in women after menopause.

A nurse is caring for a patient who has had a plaster arm cast applied. Immediately postapplication, the nurse should provide what teaching to the patient? A) The cast will feel cool to touch for the first 30 minutes. B) The cast should be wrapped snuggly with a towel until the patient gets home. C) The cast should be supported on a board while drying. D) The cast will only have full strength when dry.

Ans: D Feedback: A cast requires approximately 24 to 72 hours to dry, and until dry, it does not have full strength. While drying, the cast should not be placed on a hard surface. The cast will exude heat while it dries and should not be wrapped.

A nurse is caring for a patient who is recovering in the hospital following orthopedic surgery. The nurse is performing frequent assessments for signs and symptoms of infection in the knowledge that the patient faces a high risk of what infectious complication? A) Cellulitis B) Septic arthritis C) Sepsis D) Osteomyelitis

Ans: D Feedback: Infection is a risk after any surgery, but it is of particular concern for the postoperative orthopedic patient because of the risk of osteomyelitis. Orthopedic patients do not have an exaggerated risk of cellulitis, sepsis, or septic arthritis when compared to other surgical patients.

A patient is scheduled for a total hip replacement and the surgeon has explained the risks of blood loss associated with orthopedic surgery. The risk of blood loss is the indication for which of the following actions? A) Use of a cardiopulmonary bypass machine B) Postoperative blood salvage C) Prophylactic blood transfusion D) Autologous blood donation

Ans: D Feedback: Many patients donate their own blood during the weeks preceding their surgery. Autologous blood donations are cost effective and eliminate many of the risks of transfusion therapy. Orthopedic surgery does not necessitate cardiopulmonary bypass and blood is not salvaged postoperatively. Transfusions are not given prophylactically.

A nurse is providing discharge education to a patient who is going home with a cast on his leg. What teaching point should the nurse emphasize in the teaching session? A) Using crutches efficiently B) Exercising joints above and below the cast, as ordered C) Removing the cast correctly at the end of the treatment period D) Reporting signs of impaired circulation

Ans: D Feedback: Reporting signs of impaired circulation is critical; signs of impaired circulation must be reported to the physician immediately to prevent permanent damage. For this reason, this education is a priority over exercise and crutch use. The patient does not independently remove the cast.

A nurse is assessing the neurovascular status of a patient who has had a leg cast recently applied. The nurse is unable to palpate the patient's dorsalis pedis or posterior tibial pulse and the patient's foot is pale. What is the nurse's most appropriate action? A) Warm the patient's foot and determine whether circulation improves. B) Reposition the patient with the affected foot dependent. C) Reassess the patient's neurovascular status in 15 minutes. D) Promptly inform the primary care provider.

Ans: D Feedback: Signs of neurovascular dysfunction warrant immediate medical follow-up. It would be unsafe to delay. Warming the foot or repositioning the patient may be of some benefit, but the care provider should be informed first.

A nurse is caring for a patient who has had a total hip replacement. The nurse is reviewing health education prior to discharge. Which of the patient's statements would indicate to the nurse that the patient requires further teaching? A) "I'll need to keep several pillows between my legs at night." B) "I need to remember not to cross my legs. It's such a habit." C) "The occupational therapist is showing me how to use a 'sock puller' to help me get dressed." D) "I will need my husband to assist me in getting off the low toilet seat at home."

Ans: D Feedback: To prevent hip dislocation after a total hip replacement, the patient must avoid bending the hips beyond 90 degrees. Assistive devices, such as a raised toilet seat, should be used to prevent severe hip flexion. Using an abduction pillow or placing several pillows between the legs reduces the risk of hip dislocation by preventing adduction and internal rotation of the legs. Likewise, teaching the patient to avoid crossing the legs also reduces the risk of hip dislocation. A sock puller helps a patient get dressed without flexing the hips beyond 90 degrees.

A patient with a fractured femur is in balanced suspension traction. The patient needs to be repositioned toward the head of the bed. During repositioning, what should the nurse do? A) Place slight additional tension on the traction cords. B) Release the weights and replace them immediately after positioning. C) Reposition the bed instead of repositioning the patient. D) Maintain consistent traction tension while repositioning.

Ans: D Feedback: Traction is used to reduce the fracture and must be maintained at all times, including during repositioning. It would be inappropriate to add tension or release the weights. Moving the bed instead of the patient is not feasible.

A nurse is providing an educational class to a group of older adults at a community senior center. In an effort to prevent osteoporosis, the nurse should encourage participants to ensure that they consume the recommended adequate intake of what nutrients? Select all that apply. A) Vitamin B12 B) Potassium C) Calcitonin D) Calcium E) Vitamin D

Ans: D, E Feedback: A diet rich in calcium and vitamin D protects against skeletal demineralization. Intake of vitamin B12and potassium does not directly influence the risk for osteoporosis. Calcitonin is not considered to be a dietary nutrient.

Which medication classification is prescribed when allergy is a factor causing the skin disorder? a) Antihistamines b) Corticosteroids c) Local anesthetics d) Antibiotics

Antihistamines Explanation: Antihistamines are frequently prescribed when an allergy is a factor in causing the skin disorder. They relieve itching and shorten the duration of allergic reaction. Corticosteroids are used to relieve inflammatory or allergic symptoms. Antibiotics are used to treat infectious disorders. Local anesthetics are used to relieve minor skin pain and itching.

A physician writes an order to discontinue skeletal traction on an orthopedic patient. The nurse should anticipate what subsequent intervention? A) Application of a walking boot B) Application of a cast C) Education on how to use crutches D) Passive range of motion exercises

B After skeletal traction is discontinued, internal fixation, casts, or splints are then used to immobilize and support the healing bone. The use of a walking boot, crutches, or ROM exercises could easily damage delicate, remodeled bone.

A patient has been diagnosed with osteomalacia. What common symptoms does the nurse recognize correlate with the diagnosis? a. Bone fractures and kyphosis b. Bone pain and tenderness c. Muscle weakness and spasms d. Softened and compressed vertebrae

B. Osteomalacia is a metabolic bone disease characterized by inadequate mineralization of bone. As a result, the skeleton softens and weakens, causing pain, tenderness to touch, bowing of the bones, and pathologic fractures.

A patient has stepped in a hole in the yard, causing an ankle injury. The ankle is edematous and painful to palpation. How long should the nurse inform the patient that the acute inflammatory stage will last? a. Less than 24 hours b. Between 24 and 48 hours c. About 72 hours d. At least 1 week

Between 24 and 48 hours

Traumatic dislocations are orthopedic emergencies. Why?

because the associated joint structures, blood supply, and nerves are displaced and may be entrapped with extensive pressure on them.

___________, secreted by the thyroid gland in response to elevated blood calcium levels, inhibits bone resorption and increases the deposit of calcium in bone

Calcitonin

Which medication directly inhibits osteoclasts thereby reducing bone loss and increasing BMD? a) Calcitonin (Miacalcin) b) Vitamin D c) Raloxifene (Evista) d) Teriparatide (Forteo)

Calcitonin (Miacalcin) Explanation: Miacalcin directly inhibits osteoclasts, thereby reducing bone loss and increased BMD. Evista reduces the risk of osteoporosis by preserving BMD without estrogenic effects on the uterus. Forteo has been recently approved by the FDA for the treatment of osteoporosis.

A home care nurse assesses for disease complications in a client with bone cancer. Which laboratory value may indicate the presence of a disease complication? a) Sodium level of 110 mEq/L b) Calcium level of 11.6 mg/dl c) Magnesium level of 0.9 mg/dl d) Potassium level of 6.3 mEq/L

Calcium level of 11.6 mg/dl Explanation: In clients with bone cancer, tumor destruction of bone commonly causes excessive calcium release. When the calcium-excreting capacity of the kidneys and GI tract is exceeded, the serum calcium level rises above normal, leading to hypercalcemia (a serum calcium level greater than 10.2 mg/dl). Hyperkalemia (a potassium level greater than 5 mEq/L) isn't caused by bone cancer and is seldom associated with chemotherapy. Hyponatremia (a sodium level less than 135 mEq/L) and hypomagnesemia (a magnesium level less than 1.3 mg/dl) are potential adverse effects of chemotherapy; these electrolyte disturbances don't result directly from bone cancer.

When performing a physical assessment of a client, the client reports numbness, tingling, and pain when the nurse percusses lightly over the median nerve. The nurse recognizes that this finding is consistent with: a) Dupuytren's contracture b) Impingement syndrome c) Morton's neuroma d) Carpal tunnel syndrome

Carpal tunnel syndrome Explanation: Tinel's sign (numbness, tingling, and pain in response to light percussion over the median nerve) is a positive finding for carpal tunnel syndrome.

Which group is at the greatest risk for osteoporosis? a) Men b) African American women c) Asian women d) Caucasian women

Caucasian women Explanation: Small-framed, nonobese Caucasian women are at greatest risk for osteoporosis. Asian women of slight build are at risk for low peak BMD. African American women, who have a greater bone mass that Caucasian women, are less susceptible to osteoporosis. Men have a greater peak bone mass and do not experience sudden estrogen reduction.

When an infection is blood borne the manifestations include which of the following symptoms? a) Hypothermia b) Hyperactivity c) Chills d) Bradycardia

Chills Explanation: Manifestations include chills, high fever, rapid pulse, and generalized malaise.

pin site care ____________2 mg/mL solution is the most effective cleansing solution.

Chlorhexidine *If chlorhexidine is contraindicated (due to known sensitivity or skin reaction), saline solution should be used for cleansing.

To help prevent osteoporosis, what should a nurse advise a young woman to do? a) Consume at least 1,000 mg of calcium daily. b) Keep the serum uric acid level within the normal range. c) Encourage the use of a firm mattress. d) Avoid trauma to the affected bone.

Consume at least 1,000 mg of calcium daily. Explanation: To help prevent osteoporosis, the nurse should encourage the client to consume at least the recommended daily allowance (RDA) of calcium. Before menopause, the RDA is 1,000 mg; after menopause, it's 1,500 mg. Because osteoporosis affects all bones, avoiding trauma to the affected bone only is inappropriate. Using a firm mattress and keeping the uric acid level within the normal range don't relate to osteoporosis. The nurse should encourage a client with ankylosing spondylitis to sleep on a firm mattress. The nurse should advise a client with gouty arthritis to keep the serum uric acid level in the normal range.

An Electromyography is usually contraindicated in patients receiving anticoagulant therapy (e.g., warfarin) WHY?

because the needle electrodes may cause bleeding within the muscle.

When describing malignant bone tumors to a group of students, which of the following would the instructor cite as the usual location? a) Distal femur around the knee b) Wrist-hand junction c) Proximal humerus d) Femur-hip area

Distal femur around the knee Explanation: Malignant bone tumors usually are located around the knee in the distal femur or proximal fibula; a few are found in the proximal humerus. The wrist-hand junction and femur-hip area are not common sites.

Muscle fatigue is thought to be caused by depletion of ________ and accumulation of lactic acid. As a result, the cycle of muscle contraction and relaxation cannot continue

glycogen

Stored muscle _________ is used to supply glucose during periods of activity.

glycogen

Which of the following clinical manifestations would the nurse expect to find in a client who has had osteoporosis for several years? a) Diarrhea b) Decreased height c) Bone spurs d) Increased heel pain

Decreased height Explanation: Clients with osteoporosis become shorter over time.

RANKL Inhibitor

Denosumab (Prolia) - Monoclonal antibody that increases BMD and reduces the porosity of cortical bone by inhibiting the effects of TNF on osteoclasts, inhibiting their activity

The client diagnosed with osteosarcoma is scheduled for a surgical amputation. Which nursing diagnosis would be a priority for this client compared with other surgical clients? a) Impaired physical mobility b) Risk for infection c) Inadequate nutrition d) Disturbed body image

Disturbed body image Explanation: Amputation of a body part can result in disturbances in body image.

A patient comes to the clinic reporting low back pain radiating down the left leg. After diagnostic studies rule out any pathology, the physician prescribes a serotonin-norepinephrine reuptake inhibitor (SNRI). Which medication does the nurse anticipate educating the patient about? a. Amitriptyline b. Duloxetine c. Gabapentin d. Cyclobenzaprine

Duloxetine

The nurse is caring for a patient with bone metastasis from a primary breast cancer. The patient reports muscle weakness and nausea and is voiding large amounts frequently. Cardiac dysrhythmias are observed on the telemetry monitor. What should the nurse suspect based on these clinical manifestations? a. Hypercalcemia b. Hypocalcemia c. Hypokalemia d. Hyperkalemia

Hypercalcemia

Why is osteoporosis more common in women?

less estrogen * estrogen increases osteoblast and inhibits osteoclast

During a routine physical examination of a client, the nurse observes a flexion deformity of the promixal interphalangeal (PIP) joint of two toes on the right foot. The nurse documents this finding as which of the following? a) Bunion b) Mallet toe c) Hallux valgus d) Hammer toe

Hammer toe Explanation: Hammer toe is a flexion deformity of the proximal interphalangeal (PIP) joint and may involve several toes. Mallet toe is a flexion deformity of the distal interphalangeal joint (DIP), and also can affect several toes. Hallux valgus, also called a bunion, is a deformity of the great (large) toe at its metatarsophalangeal joint.

Ms. Simpson has come to the clinic with foot pain. The physician has described her problem as a flexion deformity of the proximal interphalangeal joint. What is the name of this disorder? a) Mallet toe b) Hammer toe c) Heberden's nodes d) Hallux valgus (bunion)

Hammer toe Explanation: Hammer toe is a flexion deformity of the proximal interphalangeal joint. Mallet toe is a flexion deformity of the distal interphalangeal joint. Bunion is a deformity of the great toe at its metatarsophalangeal joint. Heberden's nodes are bony enlargements of the distal interphalangeal joints. This is a finding in degenerative joint disease.

A patient has been prescribed alendronate (Fosamax) for the prevention of osteoporosis. Which of the following is the highest priority nursing intervention associated with the administration of the medication? a) Have patient sit upright for 60 minutes following administration b) Encourage patient to get yearly dental exams c) Assess for the use of corticosteroids d) Ensure adequate intake of vitamin D in the diet

Have patient sit upright for 60 minutes following administration Explanation: While all interventions are appropriate, the highest priority is having the patient sit upright for 60 minutes following the administration of the medication. This will prevent irritation and potential ulceration of the esophagus. The patient should have adequate intake of vitamin D and obtain yearly dental exams. The concurrent use of corticosteroids and Fosamax is link to a complication of osteonecrosis.

A physician prescribes raloxifene (Evista) to a hospitalized patient. The patient's history includes a right hip fracture, hysterectomy, deep vein thrombosis, and hypertension. Which of the following actions by the nurse demonstrates safe nursing care? a) Administering the raloxifene (Evista) with food or milk b) Having the patient sit upright for 30-60 minutes following administration c) Holding the raloxifene (Evista) and notifying the physician d) Administering the raloxifene (Evista) in the evening

Holding the raloxifene (Evista) and notifying the physician Explanation: Raloxifene (Evista) is contraindicated in patients with a history of deep vein thrombosis. The nurse should hold the medication and notify the physician. Raloxifene (Evista) can be given without regard to food or time of day. Raloxifene (Evista) is a selective estrogen receptor modulation (SERM) medication. Sitting upright for 30-60 minutes is for the classification of bisphosphonates.

normal capillary refill time

less than 2 seconds

___________ usually occurs from repetitive overhead movement of the arm or from acute trauma resulting in irritation and eventual inflammation of the rotator cuff tendons or the subacromial bursa as they grate against the coracoacromial arch

Impingement

Which of the following aspects should a nurse include in the teaching plan for a patient with osteomalacia? a) Include the supplements of calcium, phosphorus, and vitamin D b) Avoid any activity or exercise c) Avoid dairy products d) Avoid green leafy vegetables

Include the supplements of calcium, phosphorus, and vitamin D Explanation: The nurse should encourage the patients with osteomalacia to include the supplements of calcium, phosphorus, and vitamin D; adequate nutrition; exposure to sunlight; and progressive exercise and ambulation. Patients need not avoid dairy products, leafy vegetables or mild exercise.

The Fracture Risk Assessment Tool (FRAX®)

It is a tool that predicts a patient's 10-year risk of fracturing a hip or other major bone, which includes the spine, forearm, or shoulder

Which assessment findings would the nurse expect in a client diagnosed with osteomyelitis? a) Leukocytosis and localized bone pain b) Thrombocytopenia and ecchymosis c) Petechiae over the chest and abnormal ABGs d) Pruritus and uremic frost

Leukocytosis and localized bone pain Explanation: Clinical manifestations of osteomyelitis include signs and symptoms of sepsis and localized infection.

__________________ is a noninvasive imaging technique that uses magnetic fields and radio waves to create high-resolution pictures of bones and soft tissues. It can be used to visualize and assess torn muscles, ligaments, and cartilage; herniated discs; and a variety of hip or pelvic conditions. The patient does not experience any pain during the procedure.

Magnetic resonance imaging (MRI)

Which of the following classic symptoms would the nurse assess for to detect the development of plantar fasciitis? a) Shortening of affected leg b) Elevated temperature c) Morning heel pain d) Shortened height

Morning heel pain Explanation: Plantar fasciitis is characterized by heel pain.

Which of the following is the most common and most fatal primary malignant bone tumor? a) Osteochondroma b) Enchondroma c) Rhabdomyoma d) Osteogenic sarcoma (osteosarcoma)

Osteogenic sarcoma (osteosarcoma) Explanation: Osteogenic sarcoma (osteosarcoma) is the most common and most often fatal primary malignant bone tumor. Benign primary neoplasms of the musculoskeletal system include osteochondroma, enchondroma, and rhabdomyoma.

ropelike bundles of collagen fibrils connecting bones

ligament

major hormonal regulators of calcium homeostasis

PTH and calcitonin

interferes with your body's normal recycling process, in which new bone tissue gradually replaces old bone tissue. Over time, the disease can cause affected bones to become fragile and misshapen

Paget Disease of the Bone

6 P's

Pain Pulse Pallor Paresthesia Paralysis Pressure

Treatment of metastatic bone cancer includes which of the following? a) Radiation b) Palliation c) Chemotherapy d) Combination chemotherapy and radiation

Palliation Explanation: The treatment of metastatic bone cancer is palliative. The therapeutic goal is to relieve the patient's pain and discomfort while promoting quality of life.

The nurse is caring for a client following foot surgery. Which nursing intervention is most important for the nurse to include in the nursing care plan? a) Administer pain medication per client request. b) Monitor vital signs every 4 hours. c) Examine surgical dressing every hour. d) Perform neuromuscular assessment every hour.

Perform neuromuscular assessment every hour. Explanation: The priority nursing intervention is to perform a neuromuscular assessment every hour. Early detection of neurological and perfusion problems is critical.

detecting fluid in knee

Perform the patellar tap test or fluid displacement test to determine the presence of fluid in the knee joint.

Pes Cavus

Pes cavus (clawfoot) refers to a foot with an abnormally high arch and a fixed equines deformity of the forefoot

how physical activity affects bone growth?

Physical activity, particularly weight-bearing activity, acts to stimulate bone formation and remodeling.

Osteosarcoma clinical features

localized bone pain that may be accompanied by a tender, palpable soft tissue mass.

A client with Paget's disease comes to the hospital and complains of difficulty urinating. The emergency department physician consults urology. What should the nurse suspect is the most likely cause of the client's urination problem? a) Benign prostatic hyperplasia b) Dehydration c) Urinary tract infection (UTI) d) Renal calculi

Renal calculi Explanation: Renal calculi commonly occur with Paget's disease, causing pain and difficulty when urinating. A UTI commonly causes fever, urgency, burning, and hesitation with urination. Benign prostatic hyperplasia is common in men older than age 50; however, because the client has Paget's disease, the nurse should suspect renal calculi, not benign prostatic hyperplasia. Dehydration causes a decrease in urine production, not a problem with urination.

Which of the following terms refers to disease of a nerve root? a) Radiculopathy b) Involucrum c) Sequestrum d) Contracture

Radiculopathy Explanation: When the patient reports radiating pain down the leg, he or she is describing radiculopathy. Involucrum refers to new bone growth around the sequestrum. Sequestrum refers to dead bone in an abscess cavity. Contracture refers to abnormal shortening of muscle or fibrosis of joint structures.

The hospice nurse is assigned to care for a patient with metastatic bone cancer who wants to remain at home. What is the therapeutic goal in the care of this patient? a. Prevent the patient from having to go to the hospital for care b. Relieve pain and discomfort while promoting quality of life c. Increase the activity level of the patient to prevent complications related to immobility d. Ensure that the family accepts the patient's imminent death

Relieve pain and discomfort while promoting quality of life

Most cases of osteomyelitis are caused by which of the following microorganisms? a) Proteus species b) Escherichia coli c) Pseudomonas species d) Staphylococcus

Staphylococcus Explanation: Staphylococcus aureus causes 70% to 80% of bone infections. Proteus species are frequently found in osteomyelitis, but they do not cause the majority of bone infections. Pseudomonas species are frequently found in osteomyelitis, but they do not cause most bone infections. While E. coli is frequently found in osteomyelitis, it does not cause the majority of bone infections.

A patient stepped on an acorn while walking barefoot in the backyard and developed an infection progressing to osteomyelitis. What microorganism does the nurse recognize is most often the cause of the development of osteomyelitis? a. Proteus b. Pseudomonas c. Salmonella d. Staphylococcus aureus

Staphylococcus aureus

A client is diagnosed with osteomyelitis. This is most commonly caused by which of the following? a) Staphylococcus aureus b) Proteus vulgaris c) Escherichia coli d) Psuedomonas aeruginosa

Staphylococcus aureus Explanation: Staphylococcus aureus causes over 50% of bone infections. Other organisms include Proteus vulgaris and Pseudomonas aeruginosa, as well as E. coli.

A patient reports experiencing low back pain. What position can the nurse suggest to relieve this discomfort? a. High Fowler's to allow for maximum hip flexion b. Supine, with the knees slightly flexed and the head of the bed elevated 30 degrees c. Prone, with a pillow under the shoulders d. Supine, with the bed flat and a firm mattress in place

Supine, with the knees slightly flexed and the head of the bed elevated 30 degrees

In chronic osteomyelitis, antibiotics are adjunctive therapy in which of the following situations? a) Wound packing b) Surgical debridement c) Vitamin supplements d) Wound irrigation

Surgical debridement Explanation: In chronic osteomyelitis, antibiotics are adjunctive therapy to surgical debridement.

A client has been treated for migraine headaches for several months and comes to the clinic stating he is getting no better. The nurse is talking with the client and hears an audible click when the client is moving his jaw. What does the nurse suspect may be happening? a) Dislocated jaw b) Temporomandibular disorder c) Trigeminal neuralgia d) Loose teeth

Temporomandibular disorder Explanation: The disorder can be confused with trigeminal neuralgia and migraine headaches. The client experiences clicking of the jaw when moving the joint, or the jaw can lock, which interferes with opening the mouth. Loose teeth will not cause a clicking of the jaw. The client does not have a dislocated jaw.

A nurse is caring for a client who's experiencing septic arthritis. This client has a history of immunosuppressive therapy and his immune system is currently depressed. Which assignment is the most appropriate for the nurse caring for this client? a) The nurse caring for this client is also caring for four other immunosuppressed clients on the medical floor. b) The nurse caring for this client is also caring for four clients receiving chemotherapy for cancer treatment on the oncology floor. c) The nurse is caring for this client on the intensive care unit. d) The nurse caring for this client is also caring for two other immunosuppressed clients on the medical intensive care unit.

The nurse is caring for this client on the intensive care unit. Explanation: This client is critically ill; his diagnosis and immunosuppression place him at a high risk for infection. The most appropriate place for this client is in an intensive care unit, where the nurse can focus exclusively on his health promotion. This client shouldn't be on the oncology floor. This client requires close monitoring. The nurse caring for this client shouldn't also be caring for other clients who may require frequent interventions

A nurse is teaching a female client about preventing osteoporosis. Which teaching point is correct? a) Obtaining the recommended daily allowance of calcium requires taking a calcium supplement. b) To prevent fractures, the client should avoid strenuous exercise. c) Obtaining an X-ray of the bones every 3 years is recommended to detect bone loss. d) The recommended daily allowance of calcium may be found in a wide variety of foods.

The recommended daily allowance of calcium may be found in a wide variety of foods. Explanation: Premenopausal women require 1,000 mg of calcium per day. Postmenopausal women require 1,500 mg per day. Clients usually can get the recommended daily requirement of calcium by eating a varied diet. Osteoporosis doesn't show up on ordinary X-rays until 30% of bone has been lost. Bone densitometry, however, can detect bone loss of 3% or less. This test is sometimes recommended routinely for women older than 35 who are at risk for osteoporosis. Strenuous exercise won't cause fractures. Although supplements are available, they aren't always necessary.

- ______sign may be elicited in patients with carpal tunnel syndrome by percussing lightly over the median nerve, located on the inner aspect of the wrist. - If the patient reports tingling, numbness, and pain, the test for Tinel sign is considered positive.

Tinel

Describe the assessment technique used for Tinel sign

Tinel sign may be elicited in patients with carpal tunnel syndrome bypercussing lightly over the median nerve, located on the inner aspect of thewrist. If the patient reports tingling, numbness, and pain, the test for Tinelsign is considered positive.

Which sign may be helpful in identifying carpal tunnel syndrome? a) Kernig's b) Brudzinski's c) Babinski's d) Tinel's

Tinel's Explanation: Tinel's sign may be used to help identify carpal tunnel syndrome. The presence of the Babinski's sign can identify disease of the brain and spinal cord in adults and also exists as a primitive reflex in infants. The Brudzinski's and Kernig's sign are indicative of meningeal irritation.

Risedronate (Actonel)

Treatment of osteoporosis in men, and in women and men taking corticosteroids

The nurse teaches the patient with a high risk for osteoporosis about risk-lowering strategies including which of the following actions? a) Walk or perform weight-bearing exercises outdoors b) Decrease the intake of vitamin A and D c) Increase fiber in the diet d) Reduce stress

Walk or perform weight-bearing exercises outdoors Explanation: Risk-lowering strategies for osteoporosis include walking or exercising outdoors, performing a regular weight-bearing exercise regimen, increasing dietary calcium and vitamin D intake, smoking cessation, and consuming alcohol and caffeine consumption in moderation.

A nurse is planning discharge teaching regarding exercise for a client at risk for osteoporosis. Which of the following exercises would be appropriate? a) Bicycling b) Walking c) Yoga d) Swimming

Walking Explanation: Weight-bearing exercises should be incorporated into the client's lifestyle activities.

The nurse is assisting a client with removing shoes prior to an examination and observes that the client has a flexion deformity of several toes on both feet of the proximal interphalangeal (PIP) joints. What can the nurse encourage the client to do? a) Wear properly fitting shoes. b) Do active range of motion on the toes. c) Have surgery to fix them. d) Bind the toes so that they will straighten.

Wear properly fitting shoes. Explanation: Hammer toe is a flexion deformity of the PIP joint and may involve several toes and may result from wearing poorly fitting shoes. They will not straighten by binding the toes or doing active range of motion exercises. Surgery is an option but should be discussed with an orthopedic surgeon or podiatrist.

Volkmann's ischemic contracture

a permanent shortening (contracture) of forearm muscles, usually resulting from injury, that gives rise to a clawlike deformity of the hand, fingers, and wrist.

traction

a pull to the arm or leg muscles to bring a bone back into place when it is dislocated or fractured

cast syndrome

a rare cause of small bowel obstruction caused by compression of third part of duodenum from narrowing of the angle between superior mesenteric artery and abdominal aorta resulting in symptoms of duodenal outflow obstruction.

Volkmann contracture

a serious complication of impaired circulation in thearm. Contracture of the fingers and wrist occurs as the result of obstructedarterial blood flow to the forearm and the hand. The patient is unable toextend the fingers, describes abnormal sensation, and exhibits signs ofdiminished circulation to the hand. Permanent damage develops within a fewhours if action is not taken

fasciotomy

a surgical incision through the fascia to relieve tension or pressure *compartment syndrome

The client is scheduled for a meniscectomy of the right knee. The nurse would plan postoperative care based on what surgical procedure? a. Excision of damaged joint fibrocartilage b. Replacement of one of the articular surfaces of a joint c. Incision and diversion of the muscle fascia d. Replacement of knee with artificial joint

a. The most common site for meniscectomy is the knee; the procedure refers to the excision of damaged joint fibrocartilage.

A patient has a cast removed after bone healing takes place. What should the nurse educate the patient to do after removal? (Select all that apply.) a. Apply an emollient lotion to soften the skin b. Control swelling with elastic bandages, as directed c. Gradually resume activities and exercise d. Use friction to remove dead surface skin by rubbing the area with a towel e. Use a razor to shave the dead skin off

a. Apply an emollient lotion to soften the skin b. Control swelling with elastic bandages, as directed c. Gradually resume activities and exercise

The nurse is caring for a patient with a pelvic fracture. What nursing assessment for a pelvic fracture should be included? a. Checking the urine for hematuria b. Palpating peripheral pulses in both lower extremities c. Testing the stool for occult blood d. Assessing level of consciousness e. Assessing pupillary response

a. Checking the urine for hematuria b. Palpating peripheral pulses in both lower extremities c. Testing the stool for occult blood

Three range-of-motion activities are avoided for a patient with a lower extremity amputation: ________, ________, and ________.

abduction, external rotation, flexion

a patient experiences sudden posterior ankle pain with an audible "pop," difficulty in plantar flexion, and inability to fully bear weight on the affected leg

achilles tendon rupture

Gliding joints

allow for limited movement in all directions and are represented by the joints of the carpal bones in the wrist.

Saddle joints

allow movement in two planes at right angles to each other. *The joint at the base of the thumb is a saddle, biaxial joint.

An artificial joint for total hip replacement involves an implant that consists of ________, ________, and ________.

an acetabular socket, a femoral shaft, a spherical bal

synarthrosis

an immovable joint

corn on foot

area of hyperkeratosis (overgrowth of a horny layer of epidermis) produced by internal pressure (the underlying bone is prominent because of a congenital or acquired abnormality, commonly arthritis) or external pressure (ill-fitting shoes).

most accurate diagnostic modality for rotator cuff tear?

arthroscopic

Common pulmonary complications for the older adult patient following a hip fracture include ________ and ________.

atelectasis, pneumonia

Instructions for the patient with low back pain include that when lifting the patient should a) bend the knees and loosen the abdominal muscles. b) avoid overreaching. c) place the load away from the body. d) use a narrow base of support.

avoid overreaching. Explanation: Instructions for the patient with low back pain should include that when lifting, the patient should avoid overreaching. The patient should also keep the load close to the body, bend the knees and tighten the abdominal muscles, use a wide base of support, and use a back brace to protect the back. When lifting, the patient with low back pain should keep the load close to the body. When lifting, the patient with low back pain should bend the knees and tighten the abdominal muscles.

Bisphosphonates are given to treat ___________

osteoporosis

most prevalent bone disease in the world?

osteoporosis

If large amounts of fluid are present in the joint spaces beneath the patella, it may be identified by assessing for the _______________ and for ballottement of the knee

balloon sign

Bone infections are more difficult to eradicate than soft tissue infections . WHY?

because bone is mostly avascular and less accessible to the body's natural immune response

The nurse must never ignore complaints of pain from the patient in a cast. Why?

because of the possibility of problems, such as impaired tissue perfusion, compartment syndrome or pressure ulcer formation.

Unintentional injuries are commonly called accidents; however, this term is considered inaccurate by trauma professionals. Why?

because the term accident infers that there is no potential for prevention; yet, health care professionals understand that prevention plays a major role in decreasing the rate of unintentional injuries.

Hematopoiesis

blood cell formation; occurs in red bone marrow

Consequence of osteoporosis

bone fracture

The major consequence of osteoporosis is ________.

bone fracture

Splinting

bone immobilization by application of an orthopedic device to the injured body part

A deficiency of vitamin D results in

bone mineralization deficit, deformity, and fracture

A _________ may accurately identify bone disease before it can be detected on x-ray; as such, it may diagnose a stress fracture in a patient who continues to experience pain after x-ray findings are negative

bone scan

The nurse assesses for perineal nerve injury by checking the patient's casted leg for the primary symptoms of ________, ________, and ________.

burning, numbness, tingling

___________ is a sac filled with synovial fluid that cushions the movement of tendons, ligaments, and bones over bones or other joint structures

bursa

A patient was climbing a ladder, slipped on a rung, and fell on the right side of the chest. X-ray studies reveal three rib fractures, and the patient reports pain with inspiration. What is the anticipated treatment for this patient? a. Chest strapping b. Mechanical ventilation c. Coughing and deep breathing with pillow splinting d. Thoracentesis

c. Coughing and deep breathing with pillow splinting

Bone formation is enhanced by ________, ________, and ________.

calcium intake, muscular activity, weight bearing

examples of short bones

carpals and tarsals

Arthrocentesis

carried out to obtain synovial fluid for purposes of examination or to relieve pain due to effusion.

The ends of long bones are covered at the joints by articular ___________, which is tough, elastic, and avascular tissue

cartilage

A ____ is used to immobilize a reduced fracture, to correct or prevent a deformity (e.g., clubfoot, hip displacement), apply uniform pressure to underlying soft tissue, or support and stabilize weakened joints. Generally, it permit mobilization of the patient while restricting movement of the affected body part.

cast

Early manifestations Impingement Syndrome

characterized by edema from hemorrhage of these structures, pain, shoulder tenderness, limited movement, muscle spasm, and eventual disuse atrophy. The process may progress to a partial or complete rotator cuff tear

open fracture

compound fracture; broken bone with an open wound

Carpal tunnel syndrome

compression of the median nerve as it passes between the ligament and the bones and tendons of the wrist

is a soft tissue injury produced by blunt force, such as a blow, kick, or fall, causing small blood vessels to rupture and bleed into soft tissues (ecchymosis or bruising).

contusion

The nurse is assessing the feet of a patient and observes an overgrowth of the horny layer of the epidermis. What condition will the nurse educate the patient about? a. Bunion b. Clawfoot c. Corn d. Hammer toe

corn

sequestrum

dead bone in abscess cavity

avascular necrosis

death of tissue due to insufficient blood supply

Immobilization in a cast, splint, or brace can cause muscle atrophy and loss of strength, and can place patients at risk for _________ syndrome, which is the deterioration of body systems as a result of prescribed or unavoidable musculoskeletal inactivity

disuse syndrome

Electromyography may be contraindicated in patients with extensive skin infections. WHY?

due to the risk of spreading infection from the skin to the muscle. The nurse instructs the patient to avoid using any lotions or creams on the day of the test

When a meniscus is torn, the synovial membrane secretes additional synovial fluid due to the irritation, and the knee becomes very _____________

edematous.

Hinge joints permit movement in only two planes:

either flexion or extension (e.g., the elbow and the knee).

To help minimize calcium loss from a hospitalized client's bones, the nurse should: a) provide supplemental feedings between meals. b) provide the client dairy products at frequent intervals. c) encourage the client to walk in the hall. d) reposition the client every 2 hours.

encourage the client to walk in the hall. Explanation: Calcium absorption diminishes with reduced physical activity because of decreased bone stimulation. Therefore, encouraging the client to increase physical activity, such as by walking in the hall, helps minimize calcium loss. Turning or repositioning the client every 2 hours wouldn't increase activity sufficiently to minimize bone loss. Providing dairy products and supplemental feedings wouldn't lessen calcium loss — even if the dairy products and feedings contained extra calcium — because the additional calcium doesn't increase bone stimulation or osteoblast activity.

A nurse is caring for a female client following a motor vehicle accident resulting in paraplegia. The client is ready for discharge to home with her husband, who states, "I'm scared to carry her because I'm afraid I'll either hurt my back or drop her." A nurse identifies a need for discharge teaching of the husband in regard to: a) nutritional changes for the client with paraplegia. b) ergonomic principles and body mechanics. c) the importance of monitoring urinary elimination. d) signs and symptoms of chronic back pain that he should report to his physician.

ergonomic principles and body mechanics. Explanation: The husband's statement indicates a need for teaching in regard to client mobility and transfer techniques. Although urinary elimination, nutrition, and pain are components of care for clients with paraplegia, education about ergonomic principles and body mechanics is most appropriate at this time based on the husband's statement.

bursa

fluid-filled sac found in connective tissue, usually in the area of joints

Skin traction. Care must be taken to avoid pressure on the peroneal nerve at the point at which it passes around the neck of the fibula just below the knee when traction is applied to the lower extremity. Pressure at this point can cause __________

footdrop.

Diarthrosis joints are

freely movable

Arthrodesis

fusion of a joint (most often the wrist or knee) for stabilization and pain relief.

Benefits of external fixation,

immediate fracture stabilization, minimization of blood loss (as compared touse of internal fixation), increased patient comfort, improved wound care, and promotion of early mobilization and weight-bearing on the affected limb, and active exercise of adjacent uninvolved joints.

if patient with a cast has severe burning pain over bony prominences, especially the heels, anterior ankles, and elbows, warns of an _________________________

impending pressure ulcer.

What is impingement syndrome and what measures are necessary to promote shoulder healing?

impingement syndrome is a general term that describes impaired movement of the rotator cuff of the shoulder. Impingement usually occurs from repetitive overhead movement of the arm or from acute trauma resulting in irritation and eventual inflammation of the rotator cuff tendons or the subacromial bursa as they grate against the coracoacromial arch

Bisphosphonates Side effects

include gastrointestinal symptoms, including dyspepsia, nausea, flatulence, diarrhea, and constipation

Excessive thyroid hormone production in adults (e.g., Graves' disease) can result in __________ bone resorption and decreased bone formation. Increased levels of cortisol have these same effects

increased

During _____________ exercises, the muscle doesn't noticeably change length and the affected joint doesn't move. These exercises help maintain strength.

isometric

osteocyte

mature bone cell

Unrelieved pain for a patient in a cast must be immediately reported to avoid ________, ________, ________, and ________

necrosis, impaired tissue perfusion, pressure ulcer formation, possibleparalysis

process in which minerals (calcium) are deposited in bone matrix

ossification

What is the bone forming cell?

osteoblast

Estrogen stimulates __________ and inhibits ______________; therefore, bone formation is enhanced and resorption is inhibited.

osteoblasts osteoclasts

in Primary osteoporosis. Failure to develop optimal peak bone mass and low vitamin D levels contribute to the development of _____________ without associated bone loss.

osteopenia

people who are unable to engage in regular weight-bearing activities, such as those on prolonged bed rest or those with some physical disabilities, have increased bone resorption from calcium loss, and their bones become ______________ (reduced in terms of mass) and weak.

osteopenic

Injury to the _________ nerve as a result of pressure is a cause of footdrop (the inability to maintain the foot in a normally flexed position). Consequently, the patient drags the foot when ambulating.

peroneal

Weakness of dorsiflexion or foot movement and inversion of the foot might indicate pressure on the common ________ nerve.

peroneal

Pivot joints allow one bone to move around a central axis without displacement. An example is:

radius and the ulna. * They permit rotation for such activities as turning a doorknob.

injury to a ligament

sprain

Compartment syndrome is managed by maintaining the extremity at the heart level (not above heart level), and opening and bivalving the cast or opening the splint, if one or the other is present.

true

Primary osteoporosis occurs in women after menopause (usually by age 51) and in men later in life, but it is not merely a consequence of aging.

true

The nurse knows to assess a patient for deep vein thrombosis (DVT) by assessing the lower extremities for: ________, ________, ________, and ________.

unilateral calf tenderness, warmth, redness, swelling

List the danger signs of possible circulatory constriction that the nurse should assess for in a casted extremity.

unrelieved pain, swelling, discoloration, tingling,numbness, inability to move fingers or toes, or any temperature changes

brace

used to provide support, control movement, and prevent additional injury. They are custom-fitted to various parts of the body; thus, they tend to be indicated for longer-term use than splints

Impingement Syndrome

usually occurs from repetitive overhead movement of the arm or from acute trauma resulting in irritation and eventual inflammation of the rotator cuff tendons or the subacromial bursa as they grate against the coracoacromial arch.


Ensembles d'études connexes

Chapter 9 - Real Estate Contracts Practice Questions

View Set

Psychology of Advertising- Chapter 12

View Set

Psych Ch. 13- Feeding and Eating Disorders

View Set

Chem Exam #2 CH 14, ExAM #2 CH 13, Chem Exam #2 Ch 15

View Set

MLT ASCP Practice Questions 3.15.16

View Set

Partnership Liquidation Lump sum

View Set